律证论坛
Would you like to react to this message? Create an account in a few clicks or log in to continue.

Quantum Field Theory III

向下

Quantum Field Theory III Empty Quantum Field Theory III

帖子 由 一星 2014-09-11, 12:31

In the present volume, we concentrate on the classical aspects of gauge theory
related to curvature. These have to be supplemented by the crucial, but elusive
quantization procedure. The quantization of the Maxwell–Dirac system leads to
quantum electrodynamics (see Vol. II). The quantization of both the full Standard
Model in elementary particle physics and the quantization of gravitation will be
studied in the volumes to come.
One cannot grasp modern physics without understanding gauge theory,
which tells us that the fundamental interactions in nature are based on
parallel transport, and in which forces are described by curvature, which
measures the path-dependence of the parallel transport.
Gauge theory is the result of a fascinating long-term development in both mathematics
and physics. Gauge transformations correspond to a change of potentials,
and physical quantities measured in experiments are invariants under gauge transformations.
Let us briefly discuss this.
Gauss discovered that the curvature of a two-dimensional su***ce is an intrinsic
property of the su***ce. This means that the Gaussian curvature of the su***ce can
be determined by using measurements on the su***ce (e.g., on the earth) without
using the surrounding three-dimensional space. The precise formulation is provided
by Gauss’ theorema egregium (the egregious theorem). Bernhard Riemann (1826–
1866) and ´Elie Cartan (1859–1951) formulated far-reaching generalizations of the
theorema egregium which lie at the heart of
• modern differential geometry (the curvature of general fiber bundles), and
• modern physics (gauge theories).
Interestingly enough, in this way,
• Einstein’s theory of general relativity (the curvature of the four-dimensional
space-time manifold), and
• the Standard Model in elementary particle physics (the curvature of a specific
fiber bundle with the symmetry group U(1) × SU(2) × SU(3))
can be traced back to Gauss’ theorema egregium.
In classical mechanics, a large class of forces can be described by the differentiation
of potentials. This simplifies the solution of Newton’s equation of motion
and leads to the concept of potential energy together with energy conservation (for
the sum of kinetic and potential energy). In the 1860s, Maxwell determined that
the computation of electromagnetic fields can be substantially simplified by introducing
potentials for both the electric and the magnetic field (the electromagnetic
four-potential).
Gauge theory generalizes this by describing forces (interactions) by the
differentiation of generalized potentials (also called connections).
The point is that gauge transformations change the generalized potentials, but not
the essential physical effects.
Physical quantities, which can be measured in experiments, have to be invariant
under gauge transformations.
Parallel to this physical situation, inmathematics the Riemann curvature tensor can
be described by the differentiation of the Christoffel symbols (also called connection
coefficients or geometric potentials). The notion of the Riemann curvature tensor
was introduced by Riemann in order to generalize Gauss’ theorema egregium to
higher dimensions. In 1915, Einstein discovered that the Riemann curvature tensor
of a four-dimensional space-time manifold can be used to describe gravitation in
the framework of the theory of general relativity.
The basic idea of gauge theory is the transport of physical information
along curves (also called parallel transport).
This generalizes the parallel transport of vectors in the three-dimensional Euclidean
space of our intuition.
In 1917, it was discovered by Levi-Civita that the study of curved manifolds
in differential geometry can be based on the notion of parallel transport of
tangent vectors (velocity vectors).
In particular, curvature can be measured intrinsically by transporting a tangent
vector along a closed path. This idea was further developed by ´Elie Cartan in
the 1920s (the method of moving frames) and by Ehresmann in the 1950s (the
connection of both principal fiber bundles and their associated vector bundles).
The very close relation between
• gauge theory in modern physics (the transport of local SU(2)-phase factors investigated
by Yang and Mills in 1954), and
• the formulation of differential geometry in terms of fiber bundles in modern
mathematics
was only noticed by physicists in 1975 (see T. Wu and C. Yang, Concept of nonintegrable
phase factors and global formulation of gauge fields, Phys. Rev. D12
(1975), 3845–3857).
The present Volume III on gauge theory and the following Volume IV on quantum
mathematics form a unified whole. The two volumes cover the following topics:

Volume III: Gauge Theory
Part I: The Euclidean Manifold as a Paradigm
Chapter 1: The Euclidean Space E3 (Hilbert Space and Lie Algebra Structure)
Chapter 2: Algebras and Duality (Tensor Algebra, Grassmann Algebra, Clifford
Algebra, Lie Algebra)
Chapter 3: Representations of Symmetries in Mathematics and Physics
Chapter 4: The Euclidean Manifold E3
Chapter 5: The Lie Group U(1) as a Paradigm in Harmonic Analysis and Geometry
Chapter 6: Infinitesimal Rotations and Constraints in Physics
Chapter 7: Rotations, Quaternions, the Universal Covering Group, and the Electron
Spin
Chapter 8: Changing Observers – A Glance at Invariant Theory Based on the
Principle of the Correct Index Picture
Chapter 9: Applications of Invariant Theory to the Rotation Group
Chapter 10: Temperature Fields on the Euclidean Manifold E3
Chapter 11: Velocity Vector Fields on the Euclidean Manifold E3
Chapter 12: Covector Fields on the Euclidean Manifold E3 and Cartan’s Exterior
Differential – the Beauty of Differential Forms
Part II: Ariadne’s Thread in Gauge Theory
Chapter 13: The Commutative Weyl U(1)-Gauge Theory and the Electromagnetic
Field
Chapter 14: Symmetry Breaking
Chapter 15: The Noncommutative Yang–Mills SU(N)-Gauge Theory
Chapter 16: Cocycles and Observers
Chapter 17: The Axiomatic Geometric Approach to Vector Bundles and Principal
Bundles
Part III: Einstein’s Theory of Special Relativity
Chapter 18: Inertial Systems and Einstein’s Principle of Special Relativity
Chapter 19: The Relativistic Invariance of the Maxwell Equations
Chapter 20: The Relativistic Invariance of the Dirac Equations and the Electron
Spin
Part IV: Ariadne’s Thread in Cohomology
Chapter 21: Exact Sequences
Chapter 22: Electrical Circuits as a Paradigm in Homology and Cohomology
Chapter 23: The Electromagnetic Field and the de Rham Cohomology.

Volume IV: Quantum Mathematics
Part I: The Hydrogen Atom as a Paradigm
Chapter 1: The Non-Relativistic Hydrogen Atom via Lie Algebra, Gauss’s Hypergeometric
Functions, von Neuman’s Functional Analytic Approach, the Weyl–
Kodaira Theory, Gelfand’s Generalized Eigenfunctions, and Supersymmetry
Chapter 2: The Dirac Equation and the Relativistic Hydrogen Atom via the Clifford
Algebra of the Minkowski Space
Part II: The Four Fundamental Forces in the Universe
Chapter 3: Relativistic Invariance and the Energy–Momentum Tensor in Classical
Field Theories
Chapter 4: The Standard Model for Electroweak and Strong Interaction in Particle
Physics
Chapter 5: Gravitation, Einstein’s Theory of General Relativity, and the Standard
Model in Cosmology
Part III: Lowest-Order Radiative Corrections in Quantum Electrodynamics (QED)
Chapter 6: Dimensional Regularization for the Feynman Propagators in QED
(Quantum Electrodynamics)
Chapter 7: The Electron in an External Electromagnetic Field (Renormalization
of Electron Mass and Electron Charge)
Chapter 8: The Lamb Shift
Part IV: Conformal Symmetry
Chapter 9: Conformal Transformations According to Gauss, Riemann, and Lichtenstein
Chapter 10: Compact Riemann Su***ces
Chapter 11: Minimal Su***ces
Chapter 12: Strings and the Graviton
Chapter 13: Complex Function Theory and Conformal Quantum Field Theory
Part V: Models in Quantum Field Theory
Part VI: Distributions and the Epstein–Glaser Approach to Perturbative Quantum
Field Theory
Part VII: Nets of Operator Algebras and the Haag–Kastler Approach to Quantum
Field Theory
Part VIII: Symmetry and Quantization – the BRST Approach to Quantum Field
Theory
Part IX: Topology, Quantization, and the Global Structure of Physical Fields
Part X: Quantum Information.

Readers who want to understand modern differential geometry and modern physics
as quickly as possible should glance at the Prologue of the present volume and at
Chaps. 13 through 17 on Ariadne’s thread in gauge theory.
Cohomology plays a fundamental role in modern mathematics and physics.
It turns out that cohomology and homology have their roots in the rules for
electrical circuits formulated by Kirchhoff in 1847.
This helps to explain why the Maxwell equations in electrodynamics are closely
related to cohomology, namely, de Rham cohomology based on Cartan’s calculus
for differential forms and the corresponding Hodge duality on the Minkowski space.
Since the Standard Model in particle physics is obtained from the Maxwell equations
by replacing the commutative gauge group U(1) with the noncommutative gauge
group U(1) × SU(2) × SU(3), it should come as no great surprise that de Rham
cohomology also plays a key role in the Standard Model in particle physics via
the theory of characteristic classes (e.g., Chern classes which were invented by
Shing-Shen Chern in 1945 in order to generalize the Gauss–Bonnet theorem for
two-dimensional manifolds to higher dimensions).
It is our goal to show that the gauge-theoretical formulation of modern physics
is closely related to important long-term developments in mathematics pioneered by
Gauss, Riemann, Poincar´e and Hilbert, as well as Grassmann, Lie, Klein, Cayley,
´Elie Cartan and Weyl. The prototype of a gauge theory in physics is Maxwell’s
theory of electromagnetism. The Standard Model in particle physics is based on the
principle of local symmetry. In contrast to Maxwell’s theory of electromagnetism,
the gauge group of the Standard Model in particle physics is a noncommutative
Lie group. This generates additional interaction forces which are mathematically
described by Lie brackets.
We also emphasize the methods of invariant theory. In terms of physics, different
observers measure different values in their experiments. However, physics does
not depend on the choice of observers. Therefore, one needs both an invariant approach
and the passage to coordinate systems which correspond to the observers, as
emphasized by Einstein in the theory of general relativity and by Dirac in quantum
mechanics. The appropriate mathematical tool is provided by invariant theory.
Acknowledgments. In 2003, J¨urgen Tolksdorf initiated a series of four International
Workshops on the state of the art in quantum field theory and the search
for a unified theory concerning the four fundamental interactions in nature. I am
very grateful to Felix Finster, Olaf M¨uller, Marc Nardmann, and J¨urgen Tolksdorf
for organizing the workshop Quantum Field Theory and Gravity, Regensburg, 2010.
The following three volumes contain survey articles written by leading experts:
F. Finster, O. M¨uller, M. Nardmann, J. Tolksdorf, and E. Zeidler (Eds.),
Quantum Field Theory and Gravity: Conceptual and Mathematical Advances
in the Search for a Unified Framework, Birkh¨auser, Basel (to appear).
B. Fauser, J. Tolksdorf, and E. Zeidler (Eds.), Quantum Field Theory –
Competitive Methods, Birkh¨auser, Basel, 2008.
B. Fauser, J. Tolksdorf, and E. Zeidler (Eds.), Quantum Gravitation:
Mathematical Models and Experimental Bounds, Birkh¨auser, Basel, 2006.
These three volumes are recommended as supplements to the material contained
in the present monograph.
一星
一星

帖子数 : 3787
注册日期 : 13-08-07

返回页首 向下

Quantum Field Theory III Empty 回复: Quantum Field Theory III

帖子 由 一星 2014-09-11, 13:39

Prologue
Geometry is the knowledge of what eternally exists.
Plato of Athen (428–348 B.C.)
He who understands geometry may understand anything in this world.
Galileo Galilei (1564–1642)
The way of people to the laws of nature are not less admirable than the
laws themselves.
Johannes Kepler (1571–1630)
In humbleness, we have to admit that if ‘number’ is a product of our
imagination, ‘space’ has a reality outside of our imagination, to which a
priori we cannot assign its laws.
Gauss (1777–1855) in a letter to Bessel, 1840
This prologue should help the reader to understand the sophisticated historical
development of gauge theory in mathematics and physics.We will not follow a strict
logical route. This will be done later on. At this point, we are going to emphasize the
basic ideas. It is our goal to show the reader how the methods of modern differential
geometry work in the case of Einstein’s theory of general relativity, which describes
the gravitational force in nature. In particular, we want to show how
• the language of physicists created by Einstein and used in most physics textbooks
(based on the use of local space-time coordinates) and
• the language of mathematicians used in modern textbooks on differential geometry
(based on the invariant – i.e., coordinate-free – formulation)
are related to each other. This should help physicists to enter modern differential
geometry. One cannot grasp modern physics without understanding gauge field
theory which tells us the following crucial facts:
• interactions in nature are based on the parallel transport of physical information;
• forces are described by curvature which measures the path-dependence of the
parallel transport.
Here, we will discuss the following points:
• an interview with the Nobel prize laureate Chen Ning Yang (born 1922) on the
history of modern gauge theory,
• Einstein’s theory of general relativity on gravitation,
• changing observers in the universe and tensor calculus,
• the Riemann curvature tensor and the beauty of Gauss’ theorema egregium,
• two fundamental variational principles in general relativity,
• symmetry and Felix Klein’s invariance principle in geometry (a glance at the
history of invariant theory in the 19th century),1
• Einstein’s principle of general relativity and invariants – the geometrization of
physics (the paradigm of higher-dimensional cartography),
• gauge transformations:
– Einstein’s gauge transformation in the theory of both special relativity and
general relativity (change of the observer),
– Dirac’s unitary gauge transformations in the Hilbert space approach to quantum
mechanics (change of the observer by changing the measurement device),
– Yang’s gauge transformation by changing the local phase factor of the wave
function,
– the U(1)-gauge transformation in classical electrodynamics and quantum electrodynamics,
– the U(1) × SU(2) gauge transformations in electroweak interaction,
– the SU(3) gauge transformations in strong interaction (quantum chromodynamics),
– the U(1) × SU(2) × SU(3) gauge transformations in the Standard Model in
particle physics,
– the conformal gauge transformations in string theory,
– ´Elie Cartan’s gauge transformations in his method of moving frames (change
of the frame),
• construction of invariants by the universal index killing principle,
• Lie’s intrinsic tangent vectors,
• ´Elie Cartan’s algebraization of calculus and infinitesimals,
• Riemann’s invariant sectional curvature and the geometric meaning of Riemann’s
curvature tensor,
• Levi-Civita’s parallel transport and the geometric meaning of the Riemann curvature
tensor,
• two fundamental approaches in differential geometry:
– Gauss’ method of symmetric tensors, and
– Cartan’s method of antisymmetric tensors,
• Yang’s matrix trick (the relation between the Einstein equations in general relativity
and the Maxwell–Yang–Mills equations), and Cartan’s calculus for matrices
with differential forms as entries,
• Cartan’s structural equations:
– local structural equations,
– global structural equations,
• partial covariant derivative and the classical Ricci calculus,
• the Lie structure behind curvature,
• the generalized Riemann curvature tensor in modern mathematics and physics,
• parallel transport of physical information and curvature,
• the modern language of fiber bundles in mathematics and physics,
• summary of typical applications,
• perspectives (instantons and gauge theory, conformal symmetry and twistors,
the Seiberg–Witten equations and the quark confinement, the Donaldson theory
for 4-dimensional manifolds, Morse theory and Floer homology, quantum cohomology,
J-holomorphic curves, Frobenius manifolds, Ricci flow and the Poincar´e
conjecture).
The classical formulas (0.13) and (0.14) on page 11 for defining the Riemann curvature
tensor via Christoffel symbols for the metric tensor are clumsy. The development
of modern differential geometry was essentially influenced by the desire of
mathematicians to get insight into the true structure of curvature. This led to a
better understanding of curvature and to far-reaching generalizations which proved
to be useful in modern physics. The basic paper in mathematics is due to:
C. Ehresmann, Les connexions infinit´esimales dans un espace fibr´e differentiable
(in French) (The infinitesimal connections in a differentiable fiber
bundle), Colloque de Topologie, Bruxelles, 1950, pp. 29–55.
Charles Ehresmann (1905–1979) based his theory on ´Elie Cartan’s work created in
the 1920s.2 The first textbook on modern differential geometry was written by:
S. Kobayashi and K. Nomizu, Foundations of Differential Geometry, Vols.
1, 2, Wiley, New York, 1963.
We also recommend:
Y. Choquet-Bruhat, C. DeWitt-Morette, and M. Dillard-Bleick, Analysis,
Manifolds, and Physics. Vol. 1: Basics; Vol. 2: 92 Applications, Elsevier,
Amsterdam, 1996.
T. Frankel, The Geometry of Physics, Cambridge University Press, Cambridge,
2004.
S. Novikov and T. Taimanov, Geometric Structures and Fields, Amer.
Math. Soc., Providence, Rhode Island, 2006.
As an introduction to the theory of general relativity based on the use of local
coordinates, we recommend the classical Lecture Notes by
P. Dirac, General Theory of Relativity, Princeton University Press, 1996
(70 pages)
together with
Ø. Grøn and S. Hervik, Einstein’s Theory of General Relativity: with Modern
Applications in Cosmology, Springer, New York, 2007.
Both the invariant formulation and the formulation in terms of local coordinates is
discussed in great detail in the classic textbook by
C. Misner, K. Thorne, and A. Wheeler, Gravitation, Freeman, San Francisco,
California, 1973.
For the sophisticated mathematical problem of solving the initial-value problem for
the Einstein equations on the gravitational field, we recommend:
P. Cru´sciel and H. Friedrich, The Einstein Equations and the Large Scale
Behavior of Gravitational Fields: 50 Years of the Cauchy Problem in General
Relativity, Birkh¨auser, Boston, 2004.
Y. Choquet–Bruhat, General Relativity and the Einstein Equations, Oxford
University Press, 2008.
As a comprehensive modern textbook, we recommend:
T. Padmanabhan, Gravitation: Foundations and Frontiers, Cambridge
University Press, 2010.
The nature of dark matter is one of the great open problems in physics. We refer
to:
G. Bertone (Ed.), Particle Dark Matter, Cambridge University Press, 2010.

An Interview with Chen Ning Yang on the History of Modern
Gauge Theory
To begin with, let us quote some parts of an interview given by the physicist Chen
Ning Yang answering the questions of Dianzhou Zhang:
Zhang: Chen Ning Yang (born 1922 in Hefei, China), one of the twentieth
century’s great theoretical physicists, shared the Nobel prize in physics
with Tsung-Dao Lee in 1957 for their joint contribution to parity nonconservation
in weak interaction. Mathematicians, however, know Yang
best for the Yang–Mills gauge field theory and the Yang–Baxter equation.
After Einstein and Dirac, Yang is perhaps the twentieth-century physicist
who has had the greatest impact on the development of mathematics . . .
While a student in Kunming (China) and Chicago, Yang was impressed
with the fact that gauge invariance determined all electromagnetic interactions.
This was known from the works in the years 1918–1929 of Weyl,
Fock, and London, and through later review papers by Pauli. But by the
1940s and the early 1950s, it played only a minor and technical role in
physics. In Chicago, Yang tried to generalize the concept of gauge invariance
to non-Abelian groups (the gauge group for electromagnetism being
the Abelian group U(1)). In analogy with Maxwell’s equations he tried
Fαβ = ∂αAβ − ∂βAα,
where Aα are matrices (α, β = 0, 1, 2, 3). As Yang pointed out later on,
“This led to a mess, and I had to give up.”
In 1954, as a visiting physicist at Brookhaven National Laboratory on Long
Island, New York, Yang returned once again to the idea of generalizing
gauge invariance. His officemate was Robert Mills, who was about to finish
his Ph.D. degree at Columbia University, New York City. Yang introduced
the idea of non-Abelian gauge field to Mills, and they decided to add a
quadratic term:
Fαβ = ∂αAβ − ∂βAα + AαAβ −AβAα. (0.1)
That cleared up the “mess” and led to a beautiful new field theory.
Zhang: Did you study gauge field theory continuously after 1954?
Yang: Yes, I did . . . In the late 1960s, I began a new formulation of gauge
field theory through the approach of non-integrable phase factors. It happened
that one semester I was teaching general relativity, and I noticed
that the formula (0.1) in gauge field theory and the formula
Quantum Field Theory III 131b19430b41385b8945ea2520c1b23e      (0.2)
with α, β, γ, δ = 0, 1, 2, 3 for the Riemann curvature tensor in Riemannian
geometry are not just similar – they are, in fact, the same if one makes
the right identification of symbols.6 It is hard to describe the thrill I felt
at understanding the point.
Zhang: Is that the first time that you realized the relation between gauge
theory and differential geometry?
Yang: I had noticed the similarity between Levi-Civita’s parallel displacement
and non-integrable phase factors in gauge fields. But the exact relationship
was appreciated by me only when I realized that the formula (0.1)
in gauge field theory and the Riemann formula (0.2) are the same. With
an appreciation of the geometrical meaning of gauge theory, I consulted
Jim Simons, a distinguished geometer, who was then the chairman of the
Mathematics Department at Stony Brooke (Long Island, New York). He
said gauge theory must be related to connections on fiber bundles. I then
that is, self-dual connections. I would be inaccurate to say after studying
mathematics for thirty years, I felt prepared to return to physics.”
Yang: In 1975, impressed with the fact that gauge fields are connections
on fiber bundles, I drove to the house of Shing-Shen Chern (1911–2004)
in El Cerrito near Berkeley (California) . . . I said I found it amazing
that gauge theory are exactly connections on fiber bundles, which the
mathematicians developed without reference to the physical world. I added
“This is both thrilling and puzzling, since you mathematicians dreamed
up these concepts out of nowhere.” Chern immediately protested “No, no.
These concepts were not dreamed up. They were natural and real.”
Zhang: The Yang–Baxter equation
A(u)B(u + v)A(v) = B(v)A(u + v)B(u)
appearing in statistical mechanics is just a simple equation for matrix functions.
Why does it have such great importance?
Yang: In the simplest situation, the Yang–Baxter equation has the form
ABA = BAB.
This is the fundamental equation of Artin (1898–1962) for the braid group.
The braid group is, of course, a record of the history of permutations. It
is not difficult to understand that the history of permutations is relevant
to many problems in mathematics and physics. Looking at the developments
of the last six or seven years, I got the feeling that the Yang–Baxter
equation is the next pervasive algebraic equation after the Jacobi identity
[A, [B,C]] + [B, [C,A]] + [C, [A,B]] = 0.
The study of the Jacobi identity has, of course, led to the whole of Lie
algebra and its relationship to Lie groups that govern symmetry in nature.
Zhang: Yang–Mills theory and the Yang–Baxter equation both figure
prominently in today’s score mathematics. One can see this by the Fields
medals awarded in 1986 and 1990. Simon Donaldson was awarded a Fields
medal at the International Congress of Mathematicians held in Berkeley
in 1986. Sir Michael Atiyah spoke on Simon Donaldson’s work: “Together
with the important work of Michael Freedman (another Fields medal winner
in 1986), Donaldson’s result implied that there exist ‘exotic’ fourdimensional
spaces which are topologically but not differentially equivalent
to the standard Euclidean four-dimensional space R4 . . . Donaldson’s
results are derived from the Yang–Mills equations of theoretical physics
which are nonlinear generalizations of Maxwell’s equations. In the Euclidean
case the solution to the Yang–Mills equations giving the absolute
minimum are of special interest and called instantons.”
There were four Fields medalists in 1990: Vladimir Drinfeld, Vaughan
Jones, Shigefumi Mori, and Edward Witten. The work of three of them
was related to the Yang–Mills equations
−D ∗ F = ∗J, DF = 0
and/or the Yang–Baxter equation (see Sect. 15.4).
tried to understand fiber-bundle theory from such books as Steenrod’s
“The Topology of Fiber Bundles,” Princeton University Press, 1951, but
I learned nothing. The language of modern mathematics is too cold and
abstract for a physicist.
Zhang: I suppose only mathematicians appreciate the mathematical language
of today.
Yang: I can tell you a relevant story. About ten years ago, I gave a talk
on physics in Seoul, South Korea. I joked “There exist only two kinds
of modern mathematics books: one which you cannot read beyond the
first page and one which you cannot read beyond the first sentence. The
Mathematical Intelligencer later reprinted this joke of mine. But I suspect
many mathematicians themselves agree with me.
Zhang: When did you understand bundle theory?
Yang: In early 1975, I invited Jim Simons to give us a series of luncheon
lectures on differential forms and bundle theory. He kindly accepted the
invitation, and we learned about de Rham’s theorem, differential forms,
patching and so on . . .
Zhang: Simon’s lecture helped Wu and Yang to write a famous paper
in 1975.7 In this paper, they analyzed the intrinsic meaning of electromagnetism,
emphasizing especially its global topological aspects. They
discussed the mathematical meaning of the Aharonov–Bohm experiment
and of the Dirac magnetic monopole. They exhibited a dictionary on the
translation of terminologies used in mathematics and physics. Half a year
later, Isadore Singer of the Massachusetts Institute of Technology (MIT,
Cambridge, Massachusetts) visited Stony Brooke and discussed these matters
with Yang at length. Singer had been an undergraduate student in
physics and a graduate student in mathematics in the 1940s. He wrote in
1985:“Thirty years later I found myself lecturing on gauge theories, beginning
with the Wu and Yang dictionary and ending with instantons,
(i) We should mention Drinfeld’s pioneering work with Yuri Manin on
the construction of instantons. These are solutions to the Yang–Mills
equations which can be thought of as having particle-like properties
of localization and size. Drinfeld’s interest in physics continued with
his investigation of the Yang–Baxter equation.
(ii) Jones opened a whole new direction upon realizing that under certain
conditions solutions of the Yang–Baxter equation could be used for
constructing invariants of links . . . The theory of quantum groups
(i.e., deformations of classical Lie groups based on non-commutative
Hopf algebras) was devised by Jimbo and Drinfeld to produce solutions
of Yang–Baxter equations.
(iii) Witten described in these terms the invariants of Donaldson and Floer
(extending the earlier ideas of Atiyah) and generalized the Jones polynomials
to the case of an arbitrary ambient three-dimensional manifold.
We note with amusement that there were complaints that the plenary lectures
at the International Congress of Mathematicians in Kyoto, 1990, were
heavily slanted toward the topics of mathematical physics: “Everywhere
we heard quantum group, quantum group, quantum group!” . . .
Yang: Many theoretical physicists are, in some ways, antagonistic to mathematics,
or at least have a tendency to downplay the value of mathematics.
I do not agree with these attitudes. I have written:8 “Perhaps of my father’s
influence, I appreciate mathematics. I appreciate the value judgement of
the mathematician, and I admire the beauty and power of mathematics:
there are ingenuity and intricacy in tactical maneuvers, and breathtaking
sweeps in strategic campaigns. And, of course, miracle of miracles, some
concepts in mathematics turn out to provide the fundamental structures
that govern the physical universe!”
In the present volume, we will show that the Yang–Mills equations generalize the
Maxwell equations in electromagnetism.

Einstein’s Theory of General Relativity on Gravitation
We set

Quantum Field Theory III 9fe9108d287a6994c9f123644a43f3ba
This completes the general theory of relativity as a logical structure. The
postulate of relativity in its most general form, which makes the space-time
coordinates meaningless parameters, leads necessarily to a certain form of
gravitational theory which explains the motion of the Perihelion of the
planet Mercury.
Anyone who has really grasped the general theory of relativity, will be
captured by its beauty. It is a triumph of the general differential calculus,
which was created by Gauss (1777–1855), Riemann (1826–1866), Christoffel
(1829–1900), Ricci-Curbastro (1853–1925), Bianchi (1856–1928), and
Levi-Civita (1873–1941).
Albert Einstein, 1915
The two fundamental Einstein equations. In 1915, motivated by the study
of classical differential geometry, Einstein based his theory of general relativity on
the Riemann curvature tensor of the four-dimensional space-time manifold M4.
The points P of M4 are called space-time points or events. Einstein’s fundamental
equations read as follows:
(i) The equation of motion for the gravitational field:
Quantum Field Theory III 9fe9108d287a6994c9f123644a43f3ba                                 (0.3)
(ii) The equation of motion for the trajectories Quantum Field Theory III 334de1ea38b615839e4ee6b65ee1b103 of celestial
bodies (e.g., planets, the sun, stars, or galaxies) and light rays:
Quantum Field Theory III 9ec4496ced7ac4c1016e1f1fd1a447f7
This equation generalizes Newton’s classical equation of motion.

(P) In terms of physics, Einstein postulated that: Physics does not depend on the
choice of observers. This is Einstein’s principle of general relativity.
(M) In terms of mathematics, Einstein’s principle of general relativity is realized
by the use of tensor calculus introduced in the second half of the 19th century.

Gauss posed the following fundamental question:
Is it possible to compute the Gaussian curvature K of a 2-dimensional
su***ce by only using measurements on the su***ce?
After a long fight, Gauss found that the answer is “yes”! He discovered the following
sophisticated formula:
K(P) =R1221(P)/g(P)                    (0.19)
where g := g11g22 − g12g12. This is the famous theorema egregium. Let us discuss
this.
The Gaussian curvature K is an intrinsic property of the 2-dimensional
su***ce; it depends on the components gαβ of the metric tensor and their
first and second partial derivatives with respect to the local coordinates.
In fact, Gauss did not explicitly use the Riemann curvature tensor, but in terms
of the modern terminology, his key formula can be written as (0.19). Concerning
cartography, the theorema egregium tells us in rigorous terms that it is impossible
to introduce geographic charts which are length preserving after rescaling. Indeed,
one can show that length preserving maps preserve the components of the metric
tensor. In turn, such maps preserve the Gaussian curvature. Finally, note that the
Gaussian curvature of the sphere is positive, but the Gaussian curvature of the
Euclidean plane vanishes.
Gauss’ theorema egregium had an enormous impact on the development of
modern differential geometry and modern physics culminating in the principle
“force equals curvature.” This principle is basic for both Einstein’s
theory of general relativity on gravitation and the Standard Model in elementary
particle physics.
In order to understand the intuitive meaning of both the components Rαβ of the
Ricci tensor and the scalar curvature R on the 2-dimensional su***ce M2, observe
that the components of the Riemann curvature tensor read as
Rαβγδ = K(gαδgβγ − gαγgβδ), α, β, γ, δ = 1, 2.
Since gαβ = gβα, we get the following symmetry properties
Rαβγδ = −Rβαγδ = −Rαβδγ = Rγδαβ
for all indices α, β, γ, δ = 1, 2. Therefore, the 2*2*2*2 = 16 components of the Riemann
curvature tensor reduce to one essential component, namely,
R1221 = K(g11g22 − g12g12).
In fact, we have R1221 = −R2121 = −R1212 = −R2112. The remaining 12 components
vanish identically. For example, it follows from Rαβγδ = −Rβαγδ that
R1112 = 0. In order to simplify notation, let us introduce an orthogonal local coordinate
system, that is, we have the special case where g12 = g21 = 0.

In his famous 1854 lecture on the foundations of geometry, Riemann described
the Riemann curvature tensor only in intuitive terms. In his 1861 paper, Riemann
published the precise analytic formula of the Riemann curvature tensor for the first
time.20 In the textbook by M. Spivak, A Comprehensive Introduction to Differential
Geometry, Vol. 2, Publish or Perish, Boston, one finds seven variants of the proof
of Riemann’s solution of the Paris Academy problem.
Riemann died in 1866 at the age of 40. His collected works fill only one volume.
But his ideas, revealing deep connections between analysis, topology, and geometry,
profoundly influenced the mathematics and physics of the 20th century. This is
described in the beautiful book by K. Maurin, The Riemann Legacy: Riemannian
Ideas in Mathematics and Physics of the 20th Century, Kluwer, Dordrecht.
The importance of conformal maps. Conformal mappings are essential for
both classifying Riemann su***ces and proving the existence of minimal su***ces
with prescribed boundary curves (the problem of Plateau (1801–1883) on soap
bubbles spanned by a metallic frame).
Conformal mappings play also a fundamental role in modern physics,
namely, in string theory and conformal quantum field theory.
The point is that the principle of critical action in string theory is invariant under
conformal mappings (which represent the gauge transformations in string theory).
In 2-dimensional conformal quantum field theory, the conformal symmetry strongly
restricts the structure of possible correlation functions (i.e., Green’s functions). Two
Riemann su***ces M and N are called conformally equivalent iff there exists a
conformal diffeomorphism
χ :M→N.
Let dimRMg denote the real dimension of the space of all compact Riemann su***ces
of genus g modulo conformal equivalence. By considering the description of
Mg by real parameters called moduli, Riemann suggested that
dimRMg = 6g −6 if g = 2, 3, . . . , dimRM1 = ∞, dimRM0 = 0. (0.22)
This was the beginning of the sophisticated theory of moduli spaces which describe
the set of given geometric (or algebraic) structures up to equivalence via symmetry
groups. The rigorous proof of theorem (0.22) can be given in the setting of
Teichm¨uller spaces.
In what follows, we will pass back to the 4-dimensional space-time manifold
M4 used in Einstein’s theory of general relativity.

Symmetry and Klein’s Invariance Principle in Geometry
Felix Klein (1849–1925) emphasized the importance of invariants in geometry.
Sophus Lie (1842–1899) discovered the importance of the linearization
principle due to Newton (1643–1727) and Leibniz (1646–1716) for constructing
invariants in differential geometry via Lie algebras and Lie
groups.
´Elie Cartan (1859–1951) combined the methods of Gauss (1777–1855) and
Riemann (1826–1866) in order to describe curvature based on the ideas
due to Klein and Lie.
Folklore
Klein’s Erlangen program and gauge theory in physics. In the 19th century,
numerous new geometries emerged in mathematics (e.g., non-Euclidean geometry
and projective geometry). Missing was a general principle for classifying geometries.
In 1869, the young German mathematician Felix Klein (1849–1925) and the young
Norwegian mathematician Sophus Lie (1842–1899) met each other in Berlin and
became close friends. Klein and Lie extensively discussed the classification problem
for geometry. They agreed that symmetry groups play a distinguished role. In his
1872 Erlangen program, Felix Klein formulated the following general principle:
Geometry is the invariant theory of transformation groups.
In physics, gauge theory corresponds to a special case of this principle:
Gauge theory studies the invariants of physical fields under both space-time
transformations and gauge transformations.
The main goal of gauge theory is the formulation of
• variational principles (principle of critical action) and
• partial differential equations (Euler–Lagrange equations)
which are invariant under both space-time transformations and gauge transformations.
Such invariant variational principles and differential equations appear in:
(a) electrodynamics (the Maxwell equations),
(b) the Standard Model in elementary particle physics,
(c) the theory of general relativity (e.g., the Standard Model in cosmology).
In this connection, our main goal is
to create a differential calculus which respects both space-time transformations
and gauge transformations.
It was the beautiful idea of ´Elie Cartan to combine curvature in differential geometry
with local symmetry. Nowadays we know that precisely this idea is basic for modern
physics, too.
A glance at the history of invariant theory. Invariant theory was created
in the 19th century by George Boole (1815–1864), James Sylvester (1814–1897),
and Arthur Cayley (1821–1895). Hermann Weyl wrote:
The theory of invariants came into existence about the middle of the nineteenth
century somewhat like Minerva: a grown-up virgin, mailed in the
shining armor of algebra, she sprang forth from Cayley’s Jovian head.
Her Athens over which she ruled and which she served as a tutelary and
beneficent goddess was projective geometry.
Cayley was a master in doing long computations and in inventing algorithms. A
brief history of invariant theory can be found in the introduction of Peter Olver’s
book: Classical Invariant Theory, Cambridge University Press, 1999. We also refer
to Felix Klein’s famous book: Development of Mathematics in the 19th Century,
Math. Sci. Press, New York, 1979.
The goal of invariant theory. We are given a mathematical object O and
a symmetry group G which transforms the object O. The final goal is to construct
G-invariants of O. That is, we are looking for quantities which are assigned to O and
which are invariant under the action of the symmetry group G. Moreover, we are
interested in determining a complete system of invariants. By definition, a system
of G invariants of O is called complete iff it uniquely determines the object O up
to symmetry operations contained in the group G.

Einstein’s Principle of General Relativity and Invariants –
the Geometrization of Physics
Einstein emphasized the importance of invariants in physics.
Folklore
In the theory of general relativity, transformations of local space-time coordinates
are called gauge transformations. Using this term, one can say that
Einstein wanted to construct his theory of general relativity in such a way
that it is gauge invariant.
In other words, starting with his philosophical principle of general relativity, Einstein
was looking for a mathematical approach which describes invariants in terms
of local coordinates. The prototype of such an approach is given by cartography.
Cartography as a paradigm. In cartography, parts of the su***ce of earth are
described by local geographic charts collected in a geographic atlas. The Euclidean
coordinates of each chart are called local coordinates of earth. Obviously, geometric
properties of the su***ce of earth do not depend on the choice of the geographic
charts, for example, the distance of two points on the su***ce of earth does not
depend on the choice of local coordinates. Geometric properties are invariants with
respect to the possible choices of local coordinates.
Intuitively spoken, Einstein looked for higher-dimensional cartography.
His friend – the mathematician Marcel Grossmann (1878–1936) – told him that
Riemann generalized Gauss’ theory of cartography to higher dimensions and that
there exists a well-developed calculus for higher-dimensional manifolds, namely,
the Ricci calculus due to Gregorio Ricci-Curbastro (1853–1925). By the help of
Grossmann, Einstein studied the Ricci calculus and he applied it to his theory of
gravitation.
The geometrization of physics. Geometry is a mathematical model for describing
both invariant geometric properties and their representation by local coordinates.
In ancient times, one only considered invariant geometric properties. The
description of geometric properties by coordinates dates back to Ren´e Descartes
(1596–1650). In 1667 Descartes published his “Discours de la m´ethode” which contains,
among a detailed philosophical investigation and its application to the sciences,
the foundation of analytic geometry (e.g., the use of Cartesian coordinates).26
Einstein geometrized gravitation in his 1915 theory of general relativity. Quantum
mechanics was geometrized by Dirac, as a unitary geometry of Hilbert spaces.
In the introduction to his book “The Principles of Quantum Mechanics,” Clarendon
Press, Oxford, 1930, the young Dirac (1902–1984) wrote:
The important things in the world appear as invariants . . . The things we
are immediately aware of are the relations of these invariants to a certain
frame of reference . . . The growth of the use of transformation theory, as
applied first to relativity and later to the quantum theory, is the essence
of the new method in theoretical physics.
Finally, note that the Standard Model in particle physics starts from a classical
field theory which is closely related to the geometry of specific fiber bundles.

Symmetry and Klein’s Invariance Principle in Geometry
Felix Klein (1849–1925) emphasized the importance of invariants in geometry.
Sophus Lie (1842–1899) discovered the importance of the linearization
principle due to Newton (1643–1727) and Leibniz (1646–1716) for constructing
invariants in differential geometry via Lie algebras and Lie
groups.
´Elie Cartan (1859–1951) combined the methods of Gauss (1777–1855) and
Riemann (1826–1866) in order to describe curvature based on the ideas
due to Klein and Lie.
Folklore
Klein’s Erlangen program and gauge theory in physics. In the 19th century,
numerous new geometries emerged in mathematics (e.g., non-Euclidean geometry
and projective geometry). Missing was a general principle for classifying geometries.
In 1869, the young German mathematician Felix Klein (1849–1925) and the young
Norwegian mathematician Sophus Lie (1842–1899) met each other in Berlin and
became close friends. Klein and Lie extensively discussed the classification problem
for geometry. They agreed that symmetry groups play a distinguished role. In his
1872 Erlangen program, Felix Klein formulated the following general principle:
Geometry is the invariant theory of transformation groups.
In physics, gauge theory corresponds to a special case of this principle:
Gauge theory studies the invariants of physical fields under both space-time
transformations and gauge transformations.
The main goal of gauge theory is the formulation of
• variational principles (principle of critical action) and
• partial differential equations (Euler–Lagrange equations)
which are invariant under both space-time transformations and gauge transformations.
Such invariant variational principles and differential equations appear in:
(a) electrodynamics (the Maxwell equations),
(b) the Standard Model in elementary particle physics,
(c) the theory of general relativity (e.g., the Standard Model in cosmology).
In this connection, our main goal is
to create a differential calculus which respects both space-time transformations
and gauge transformations.
It was the beautiful idea of ´Elie Cartan to combine curvature in differential geometry
with local symmetry. Nowadays we know that precisely this idea is basic for modern
physics, too.
A glance at the history of invariant theory. Invariant theory was created
in the 19th century by George Boole (1815–1864), James Sylvester (1814–1897),
and Arthur Cayley (1821–1895). Hermann Weyl wrote:
The theory of invariants came into existence about the middle of the nineteenth
century somewhat like Minerva: a grown-up virgin, mailed in the
shining armor of algebra, she sprang forth from Cayley’s Jovian head.
Her Athens over which she ruled and which she served as a tutelary and
beneficent goddess was projective geometry.
Cayley was a master in doing long computations and in inventing algorithms. A
brief history of invariant theory can be found in the introduction of Peter Olver’s
book: Classical Invariant Theory, Cambridge University Press, 1999. We also refer
to Felix Klein’s famous book: Development of Mathematics in the 19th Century,
Math. Sci. Press, New York, 1979.
The goal of invariant theory. We are given a mathematical object O and
a symmetry group G which transforms the object O. The final goal is to construct
G-invariants of O. That is, we are looking for quantities which are assigned to O and
which are invariant under the action of the symmetry group G. Moreover, we are
interested in determining a complete system of invariants. By definition, a system
of G invariants of O is called complete iff it uniquely determines the object O up
to symmetry operations contained in the group G.

James Sylvester (1814–1897) said in 1864:
As all roads lead to Rome so I find in my own case at least that all algebraic
inquiries, sooner or later, end at the Capitol of modern algebra over whose
shining portal is inscribed the Theory of Invariants.
Invariant theory is essential for modern physics. In the present volume we will
encounter invariant theory again and again.
Einstein’s Principle of General Relativity and Invariants –
the Geometrization of Physics
Einstein emphasized the importance of invariants in physics.
Folklore
The components  of the Riemann curvature tensor depend on the choice of
local space-time coordinates, that is, they depend on the choice of the
observer. Recall that Einstein’s principle of general relativity tells us that:
Physics is independent of the choice of the observer.
This means that proper physical quantities have to be independent of the choice of
local coordinates.

In the theory of general relativity, transformations of local space-time coordinates
are called gauge transformations. Using this term, one can say that
Einstein wanted to construct his theory of general relativity in such a way
that it is gauge invariant.
In other words, starting with his philosophical principle of general relativity, Einstein
was looking for a mathematical approach which describes invariants in terms
of local coordinates. The prototype of such an approach is given by cartography.
Cartography as a paradigm. In cartography, parts of the su***ce of earth are
described by local geographic charts collected in a geographic atlas. The Euclidean
coordinates of each chart are called local coordinates of earth. Obviously, geometric
properties of the su***ce of earth do not depend on the choice of the geographic
charts, for example, the distance of two points on the su***ce of earth does not
depend on the choice of local coordinates. Geometric properties are invariants with
respect to the possible choices of local coordinates.
Intuitively spoken, Einstein looked for higher-dimensional cartography.
His friend – the mathematician Marcel Grossmann (1878–1936) – told him that
Riemann generalized Gauss’ theory of cartography to higher dimensions and that
there exists a well-developed calculus for higher-dimensional manifolds, namely,
the Ricci calculus due to Gregorio Ricci-Curbastro (1853–1925). By the help of
Grossmann, Einstein studied the Ricci calculus and he applied it to his theory of
gravitation.
The geometrization of physics. Geometry is a mathematical model for describing
both invariant geometric properties and their representation by local coordinates.
In ancient times, one only considered invariant geometric properties. The
description of geometric properties by coordinates dates back to Ren´e Descartes
(1596–1650). In 1667 Descartes published his “Discours de la m´ethode” which contains,
among a detailed philosophical investigation and its application to the sciences,
the foundation of analytic geometry (e.g., the use of Cartesian coordinates).26
Einstein geometrized gravitation in his 1915 theory of general relativity. Quantum
mechanics was geometrized by Dirac, as a unitary geometry of Hilbert spaces.
In the introduction to his book “The Principles of Quantum Mechanics,” Clarendon
Press, Oxford, 1930, the young Dirac (1902–1984) wrote:
The important things in the world appear as invariants . . . The things we
are immediately aware of are the relations of these invariants to a certain
frame of reference . . . The growth of the use of transformation theory, as
applied first to relativity and later to the quantum theory, is the essence
of the new method in theoretical physics.
Finally, note that the Standard Model in particle physics starts from a classical
field theory which is closely related to the geometry of specific fiber bundles.


由一星于2014-09-16, 13:09进行了最后一次编辑,总共编辑了10次
一星
一星

帖子数 : 3787
注册日期 : 13-08-07

返回页首 向下

Quantum Field Theory III Empty 回复: Quantum Field Theory III

帖子 由 一星 2014-09-11, 15:47

Riemann curvature tensor

From Wikipedia, the free encyclopedia


[th]General relativity[/th][size][ltr]
In the mathematical field ofdifferential geometry, theRiemann curvature tensor, or Riemann–Christoffel tensor after Bernhard Riemann and Elwin Bruno Christoffel, is the most standard way to expresscurvature of Riemannian manifolds. It associates atensor to each point of aRiemannian manifold (i.e., it is a tensor field), that measures the extent to which the metric tensor is not locally isometric to a Euclidean space. The curvature tensor can also be defined for anypseudo-Riemannian manifold, or indeed any manifold equipped with an affine connection. It is a central mathematical tool in the theory of general relativity, the modern theory of gravity, and the curvature of spacetime is in principle observable via the geodesic deviation equation. The curvature tensor represents the tidal force experienced by a rigid body moving along a geodesic in a sense made precise by the Jacobi equation.
The curvature tensor is given in terms of the Levi-Civita connection Quantum Field Theory III Fe3a83e41074834731743ab803cd4936 by the following formula:
Quantum Field Theory III D1aa621fc7af204a1dc4a2a247129ad5
where [u,v] is the Lie bracket of vector fields. For each pair of tangent vectors uvR(u,v) is a linear transformation of the tangent space of the manifold. It is linear in u and v, and so defines a tensor. Occasionally, the curvature tensor is defined with the opposite sign.
If Quantum Field Theory III 8768fa382b45018325a2213b1e367943 and Quantum Field Theory III 44fe1a6646829debb7495a5b27160ffd are coordinate vector fields then Quantum Field Theory III 7e1a8512f323229a42f899d06f32992c and therefore the formula simplifies to
Quantum Field Theory III D493bbad067a502909d1ae33781994cc
The curvature tensor measures noncommutativity of the covariant derivative, and as such is the integrability obstruction for the existence of an isometry with Euclidean space (called, in this context, flat space). The linear transformation Quantum Field Theory III 0c4809ff81769d7213ad56e521c117f5 is also called the curvature transformation or endomorphism.
The curvature formula can also be expressed in terms of the second covariant derivative defined as:[1]
Quantum Field Theory III 4bc1eae88ab608628cfb9ef7700ef819
which is linear in u and v. Then:
Quantum Field Theory III 03e061d7d6bbf63a06049251707f4a93
Thus in the general case of non-coordinate vectors u and v, the curvature tensor measures the noncommutativity of the second covariant derivative.

[/ltr][/size]
[ltr]

Contents

  [hide[/ltr]


[size][ltr]

Geometrical meaning[edit]

Informally[edit]

Imagine walking around the bounding white line of a tennis court with a stick held out in front of you. When you reach the first corner of the court, you turn to follow the white line, but you keep the stick held out in the same direction, which means you are now holding the stick out to your side. You do the same when you reach each corner of the court. When you get back to where you started, you are holding the stick out in exactly the same direction as you were when you started (no surprise there).
Now imagine you are standing on the equator of the earth, facing north with the stick held out in front of you. You walk north up along a line of longitude until you get to the north pole. At that point you turn right, ninety degrees, but you keep the stick held out in the same direction, which means you are now holding the stick out to your left. You keep walking (south obviously – whichever way you set off from the north pole, it's south) until you get to the equator. There, you turn right again (and so now you have to hold the stick pointing out behind you) and walk along the equator until you get back to where you started from. But here is the thing: the stick is pointing back along the equator from where you just came, not north up to the pole how it was when you started!
The reason for the difference is that the su***ce of the earth is curved, whereas the su***ce of a tennis court is flat, but it is not quite that simple. Imagine that the tennis court is slightly humped along its centre-line so that it is like part of the su***ce of a cylinder. If you walk around the court again, the stick still points in the same direction as it did when you started. The reason is that the humped tennis court has extrinsic curvature but nointrinsic curvature. The su***ce of the earth, however, has both extrinsic and intrinsic curvature.
The Riemann curvature tensor is a way to capture a measure of the intrinsic curvature. When you write it down in terms of its components (like writing down the components of a vector), it consists of a multi-dimensional array of sums and products of partial derivatives (some of those partial derivatives can be thought of as akin to capturing the curvature imposed upon someone walking in straight lines on a curved su***ce).

Formally[edit]

When a vector in a Euclidean space is parallel transported around a loop, it will again point in the initial direction after returning to its original position. However, this property does not hold in the general case. The Riemann curvature tensor directly measures the failure of this in a generalRiemannian manifold. This failure is known as the non-holonomy of the manifold.
Let xt be a curve in a Riemannian manifold M. Denote by τx[sub]t[/sub] : Tx[sub]0[/sub]M → Tx[sub]t[/sub]Mthe parallel transport map along xt. The parallel transport maps are related to the covariant derivative by
Quantum Field Theory III Ce4c93c3668e2b0a413112c7b0a2f161
for each vector field Y defined along the curve.
Suppose that X and Y are a pair of commuting vector fields. Each of these fields generates a pair of one-parameter groups of diffeomorphisms in a neighborhood of x0. Denote by τtX and τtY, respectively, the parallel transports along the flows of X and Y for time t. Parallel transport of a vector Z ∈ Tx[sub]0[/sub]M around the quadrilateral with sides tYsX, −tY, −sX is given by
Quantum Field Theory III 943b9540aa96aef980d7b5457aba2b21
This measures the failure of parallel transport to return Z to its original position in the tangent space Tx[sub]0[/sub]M. Shrinking the loop by sending st → 0 gives the infinitesimal description of this deviation:
Quantum Field Theory III Bd5665bab4a99dd19eca3cd514f146d1
where R is the Riemann curvature tensor.

Coordinate expression[edit]

Converting to the tensor index notation, the Riemann curvature tensor is given by
Quantum Field Theory III D88fd0341195186d5780983b2a65dc07
where Quantum Field Theory III 2de801680a781af05f07503f030a1aef are the coordinate vector fields. The above expression can be written using Christoffel symbols:
Quantum Field Theory III 131b19430b41385b8945ea2520c1b23e
(see also the list of formulas in Riemannian geometry).
The Riemann curvature tensor is also the commutator of the covariant derivative of an arbitrary covector Quantum Field Theory III Daeb7940196c9afbe9a31e9635d5edda with itself:[2][3]
Quantum Field Theory III D46d990cd2fcc4747280bd14db5ad33f
since the connection Quantum Field Theory III 5801d58a6f3e613249d9b4b0779ba6ed is torsionless, which means that the torsion tensor Quantum Field Theory III 793d02ca4784273f73787517f7eb0439 vanishes.
This formula is often called the Ricci identity.[4] This is the classical method used by Ricci and Levi-Civita to obtain an expression for the Riemann curvature tensor.[5] In this way, the tensor character of the set of quantitiesQuantum Field Theory III Ff5a979acaf29e8d61535b0717d20afb is proved.
This identity can be generalized to get the commutators for two covariant derivatives of arbitrary tensors as follows
Quantum Field Theory III 4f93d7571fdb1e08c53f4aff922291e4
This formula also applies to tensor densities without alteration, because for the Levi-Civita (not generic) connection one gets:[4]
Quantum Field Theory III 4939b446e6c6ac82fefab8de965db5dc
It is sometimes convenient to also define the purely covariant version by
Quantum Field Theory III B794c5e934726831678f39e8e6cff476

Symmetries and identities[edit]

The Riemann curvature tensor has the following symmetries:
Quantum Field Theory III F961e3e716785734f1b83d517d7d97f2Quantum Field Theory III A3e6103487763590f960fcfde08f0084Quantum Field Theory III 2339451e026ae16eea636363387f18d2
The last identity was discovered by Ricci, but is often called the first Bianchi identity or algebraic Bianchi identity, because it is equivalent to the Bianchi identity below. (Also, if there is nonzero torsion, the first Bianchi identity becomes a differential identity of the torsion tensor.) These three identities form a complete list of symmetries of the curvature tensor, i.e. given any tensor which satisfies the identities above, one can find a Riemannian manifold with such a curvature tensor at some point. Simple calculations show that such a tensor has Quantum Field Theory III 22ee00c12e0029784d37d548e57a9687 independent components.
Yet another useful identity follows from these three:
Quantum Field Theory III 97d156941218cd29e5bf16f0a536b0bd
On a Riemannian manifold one has the covariant derivative Quantum Field Theory III C5cfe7c263fd493884a50ad3584e4390 and theBianchi identity (often called the second Bianchi identity or differential Bianchi identity) takes the form:
Quantum Field Theory III 6374a3ce737b8f4a07f1c884649bcdbb
Given any coordinate chart about some point on the manifold, the above identities may be written in terms of the components of the Riemann tensor at this point as:
Skew symmetryQuantum Field Theory III Acfd26535044e246b7699f6d1eba6cfbInterchange symmetryQuantum Field Theory III 0d31dab4ec07a955fc2d09e6a91bae2fFirst Bianchi identityQuantum Field Theory III 3aa62ef4958027fc9eb2fe4d3d551560This is often writtenQuantum Field Theory III D521c7a46d666647116f179ca8fc3023where the brackets denote the antisymmetric part on the indicated indices. This is equivalent to the previous version of the identity because the Riemann tensor is already skew on its last two indices.Second Bianchi identityQuantum Field Theory III 7ec378be2ace37f03b83ec7c6e2ba89cThe semi-colon denotes a covariant derivative. Equivalently,Quantum Field Theory III Cbab855528513cf1e7bb0a0a6096c158again using the antisymmetry on the last two indices of R.
The algebraic symmetries are also equivalent to saying that R belongs to the image of the Young symmetrizer corresponding to the partition 2+2.

Special cases[edit]

Su***ces
For a two-dimensional su***ce, the Bianchi identities imply that the Riemann tensor can be expressed as
Quantum Field Theory III 9b62e85b3d5d67a18f73945491782dfb
where Quantum Field Theory III Dbca2cfdb8f17e27d4a287e2e08934e1 is the metric tensor and Quantum Field Theory III A5f3c6a11b03839d46af9fb43c97c188 is a function called the Gaussian curvature and abc and d take values either 1 or 2. The Riemann tensor has only one functionally independent component. The Gaussian curvature coincides with the sectional curvature of the su***ce. It is also exactly half the scalar curvature of the 2-manifold, while the Ricci curvaturetensor of the su***ce is simply given by
Quantum Field Theory III 35acefac15d6bbf52956b6258eb5016aSpace forms
A Riemannian manifold is a space form if its sectional curvature is equal to a constant K. The Riemann tensor of a space form is given by
Quantum Field Theory III 0e49f20df308743da462807032d2258f
Conversely, except in dimension 2, if the curvature of a Riemannian manifold has this form for some function K, then the Bianchi identities imply that K is constant and thus that the manifold is (locally) a space form.

See also[edit]

[/ltr][/size]
[size][ltr]

Notes[edit]

[/ltr][/size][list=references]
[*]Jump up^ Lawson, H. Blaine, Jr.; Michelsohn, Marie-Louise (1989). Spin Geometry. Princeton U Press. p. 154. ISBN 0-691-08542-0.
[*]Jump up^ Synge J.L., Schild A. (1949). Tensor Calculus. first Dover Publications 1978 edition. pp. 83; 107. ISBN 978-0-486-63612-2.
[*]Jump up^ P. A. M. Dirac (1996). General Theory of Relativity. Princeton University Press. ISBN 0-691-01146-X.
[*]^ Jump up to:a b Lovelock, David; Rund, Hanno (1989) [1975]. Tensors, Differential Forms, and Variational Principles. Dover. p. 84,109. ISBN 978-0-486-65840-7.
[*]Jump up^ Ricci, Gregorio; Levi-Civita, Tullio (March 1900), "Méthodes de calcul différentiel absolu et leurs applications"Mathematische Annalen(Springer) 54 (1–2): 125–201, doi:10.1007/BF01454201
[/list]
[size][ltr]

References[edit]

[/ltr][/size]




一星
一星

帖子数 : 3787
注册日期 : 13-08-07

返回页首 向下

Quantum Field Theory III Empty 回复: Quantum Field Theory III

帖子 由 一星 2014-09-11, 15:54

General relativity

From Wikipedia, the free encyclopedia


[ltr]For a generally accessible and less technical introduction to the topic, see Introduction to general relativity.[/ltr]




[th]General relativity[/th]
Quantum Field Theory III 220px-Black_Hole_Milkyway


A simulated black hole of 10 solar masses as seen from a distance of 600 kilometers with the Milky Way in the background.

[ltr]
General relativity, or thegeneral theory of relativity, is the geometrictheory of gravitationpublished by Albert Einsteinin 1916[1] and the current description of gravitation inmodern physics. General relativity generalizes special relativity and Newton's law of universal gravitation, providing a unified description of gravity as a geometric property of spaceand time, or spacetime. In particular, the curvature of spacetime is directly related to the energy and momentumof whatever matter andradiation are present. The relation is specified by theEinstein field equations, a system of partial differential equations.
Some predictions of general relativity differ significantly from those of classical physics, especially concerning the passage of time, the geometry of space, the motion of bodies in free fall, and the propagation of light. Examples of such differences includegravitational time dilation,gravitational lensing, thegravitational redshift of light, and the gravitational time delay. The predictions of general relativity have been confirmed in all observations and experiments to date. Although general relativity is not the only relativistic theory of gravity, it is the simplest theory that is consistent with experimental data. However, unanswered questions remain, the most fundamental being how general relativity can be reconciled with the laws ofquantum physics to produce a complete and self-consistent theory ofquantum gravity.
Einstein's theory has important astrophysical implications. For example, it implies the existence of black holes—regions of space in which space and time are distorted in such a way that nothing, not even light, can escape—as an end-state for massive stars. There is ample evidence that the intense radiation emitted by certain kinds of astronomical objects is due to black holes; for example, microquasars and active galactic nuclei result from the presence of stellar black holes and black holes of a much more massive type, respectively. The bending of light by gravity can lead to the phenomenon of gravitational lensing, in which multiple images of the same distant astronomical object are visible in the sky. General relativity also predicts the existence of gravitational waves, which have since been observed indirectly; a direct measurement is the aim of projects such asLIGO and NASA/ESA Laser Interferometer Space Antenna and variouspulsar timing arrays. In addition, general relativity is the basis of currentcosmological models of a consistently expanding universe.[/ltr]

[ltr]

Contents

  [hide[/ltr]

[ltr]

History[edit]


Main articles: History of general relativity and Classical theories of gravitation[/ltr]

Quantum Field Theory III 250px-Albert_Einstein_portrait


Albert Einstein developed the theories of special and general relativity. Picture from 1921.

[ltr]
Soon after publishing thespecial theory of relativity in 1905, Einstein started thinking about how to incorporategravity into his new relativistic framework. In 1907, beginning with a simple thought experiment involving an observer in free fall, he embarked on what would be an eight-year search for a relativistic theory of gravity. After numerous detours and false starts, his work culminated in the presentation to the Prussian Academy of Science in November 1915 of what are now known as theEinstein field equations. These equations specify how the geometry of space and time is influenced by whatever matter and radiation are present, and form the core of Einstein's general theory of relativity.[2]
The Einstein field equations are nonlinear and very difficult to solve. Einstein used approximation methods in working out initial predictions of the theory. But as early as 1916, the astrophysicist Karl Schwarzschildfound the first non-trivial exact solution to the Einstein field equations, the so-called Schwarzschild metric. This solution laid the groundwork for the description of the final stages of gravitational collapse, and the objects known today as black holes. In the same year, the first steps towards generalizing Schwarzschild's solution to electrically charged objects were taken, which eventually resulted in the Reissner–Nordström solution, now associated with electrically charged black holes.[3] In 1917, Einstein applied his theory to the universe as a whole, initiating the field of relativistic cosmology. In line with contemporary thinking, he assumed a static universe, adding a new parameter to his original field equations—thecosmological constant—to reproduce that "observation".[4] By 1929, however, the work of Hubble and others had shown that our universe is expanding. This is readily described by the expanding cosmological solutions found by Friedmann in 1922, which do not require a cosmological constant. Lemaître used these solutions to formulate the earliest version of the Big Bang models, in which our universe has evolved from an extremely hot and dense earlier state.[5] Einstein later declared the cosmological constant the biggest blunder of his life.[6]
During that period, general relativity remained something of a curiosity among physical theories. It was clearly superior to Newtonian gravity, being consistent with special relativity and accounting for several effects unexplained by the Newtonian theory. Einstein himself had shown in 1915 how his theory explained the anomalous perihelion advance of the planetMercury without any arbitrary parameters ("fudge factors").[7] Similarly, a 1919 expedition led by Eddington confirmed general relativity's prediction for the deflection of starlight by the Sun during the total solar eclipse of May 29, 1919,[8] *** Einstein instantly famous.[9] Yet the theory entered the mainstream of theoretical physics and astrophysics only with the developments between approximately 1960 and 1975, now known as the golden age of general relativity.[10] Physicists began to understand the concept of a black hole, and to identify quasars as one of these objects' astrophysical manifestations.[11] Ever more precise solar system tests confirmed the theory's predictive power,[12] and relativistic cosmology, too, became amenable to direct observational tests.[13]

From classical mechanics to general relativity[edit]


General relativity can be understood by examining its similarities with and departures from classical physics. The first step is the realization that classical mechanics and Newton's law of gravity admit a geometric description. The combination of this description with the laws of special relativity results in a heuristic derivation of general relativity.[14]

Geometry of Newtonian gravity[edit]

[/ltr]

Quantum Field Theory III 220px-Elevator_gravity.svg


According to general relativity, objects in a gravitational field behave similarly to objects within an accelerating enclosure. For example, an observer will see a ball fall the same way in a rocket (left) as it does on Earth (right), provided that the acceleration of the rocket provides the same relative force.

[ltr]
At the base of classical mechanicsis the notion that a body's motion can be described as a combination of free (or inertial) motion, and deviations from this free motion. Such deviations are caused by external forces acting on a body in accordance with Newton's secondlaw of motion, which states that the net force acting on a body is equal to that body's (inertial) massmultiplied by its acceleration.[15]The preferred inertial motions are related to the geometry of space and time: in the standard reference frames of classical mechanics, objects in free motion move along straight lines at constant speed. In modern parlance, their paths are geodesics, straight world lines in curved spacetime.[16]
Conversely, one might expect that inertial motions, once identified by observing the actual motions of bodies and *** allowances for the external forces (such as electromagnetism or friction), can be used to define the geometry of space, as well as a time coordinate. However, there is an ambiguity once gravity comes into play. According to Newton's law of gravity, and independently verified by experiments such as that of Eötvösand its successors (see Eötvös experiment), there is a universality of free fall (also known as the weak equivalence principle, or the universal equality of inertial and passive-gravitational mass): the trajectory of a test body in free fall depends only on its position and initial speed, but not on any of its material properties.[17] A simplified version of this is embodied in Einstein's elevator experiment, illustrated in the figure on the right: for an observer in a small enclosed room, it is impossible to decide, by mapping the trajectory of bodies such as a dropped ball, whether the room is at rest in a gravitational field, or in free space aboard an accelerating rocket generating a force equal to gravity.[18]
Given the universality of free fall, there is no observable distinction between inertial motion and motion under the influence of the gravitational force. This suggests the definition of a new class of inertial motion, namely that of objects in free fall under the influence of gravity. This new class of preferred motions, too, defines a geometry of space and time—in mathematical terms, it is the geodesic motion associated with a specificconnection which depends on the gradient of the gravitational potential. Space, in this construction, still has the ordinary Euclidean geometry. However, spacetime as a whole is more complicated. As can be shown using simple thought experiments following the free-fall trajectories of different test particles, the result of transporting spacetime vectors that can denote a particle's velocity (time-like vectors) will vary with the particle's trajectory; mathematically speaking, the Newtonian connection is not integrable. From this, one can deduce that spacetime is curved. The result is a geometric formulation of Newtonian gravity using only covariantconcepts, i.e. a description which is valid in any desired coordinate system.[19] In this geometric description, tidal effects—the relative acceleration of bodies in free fall—are related to the derivative of the connection, showing how the modified geometry is caused by the presence of mass.[20]

Relativistic generalization[edit]

[/ltr]


[ltr]
As intriguing as geometric Newtonian gravity may be, its basis, classical mechanics, is merely a limiting case of(special) relativistic mechanics.[21] In the language of symmetry: where gravity can be neglected, physics is Lorentz invariantas in special relativity rather than Galilei invariant as in classical mechanics. (The defining symmetry of special relativity is thePoincaré group which also includes translations and rotations.) The differences between the two become significant when we are dealing with speeds approaching the speed of light, and with high-energy phenomena.[22]
With Lorentz symmetry, additional structures come into play. They are defined by the set of light cones (see the image on the left). The light-cones define a causal structure: for each event A, there is a set of events that can, in principle, either influence or be influenced by A via signals or interactions that do not need to travel faster than light (such as event B in the image), and a set of events for which such an influence is impossible (such as event C in the image). These sets are observer-independent.[23]In conjunction with the world-lines of freely falling particles, the light-cones can be used to reconstruct the space–time's semi-Riemannian metric, at least up to a positive scalar factor. In mathematical terms, this defines aconformal structure.[24]
Special relativity is defined in the absence of gravity, so for practical applications, it is a suitable model whenever gravity can be neglected. Bringing gravity into play, and assuming the universality of free fall, an analogous reasoning as in the previous section applies: there are no global inertial frames. Instead there are approximate inertial frames moving alongside freely falling particles. Translated into the language of spacetime: the straight time-like lines that define a gravity-free inertial frame are deformed to lines that are curved relative to each other, suggesting that the inclusion of gravity necessitates a change in spacetime geometry.[25]
A priori, it is not clear whether the new local frames in free fall coincide with the reference frames in which the laws of special relativity hold—that theory is based on the propagation of light, and thus on electromagnetism, which could have a different set of preferred frames. But using different assumptions about the special-relativistic frames (such as their being earth-fixed, or in free fall), one can derive different predictions for thegravitational redshift, that is, the way in which the frequency of light shifts as the light propagates through a gravitational field (cf. below). The actual measurements show that free-falling frames are the ones in which light propagates as it does in special relativity.[26] The generalization of this statement, namely that the laws of special relativity hold to good approximation in freely falling (and non-rotating) reference frames, is known as the Einstein equivalence principle, a crucial guiding principle for generalizing special-relativistic physics to include gravity.[27]
The same experimental data shows that time as measured by clocks in a gravitational field—proper time, to give the technical term—does not follow the rules of special relativity. In the language of spacetime geometry, it is not measured by the Minkowski metric. As in the Newtonian case, this is suggestive of a more general geometry. At small scales, all reference frames that are in free fall are equivalent, and approximately Minkowskian. Consequently, we are now dealing with a curved generalization of Minkowski space. The metric tensor that defines the geometry—in particular, how lengths and angles are measured—is not the Minkowski metric of special relativity, it is a generalization known as a semi- orpseudo-Riemannian metric. Furthermore, each Riemannian metric is naturally associated with one particular kind of connection, the Levi-Civita connection, and this is, in fact, the connection that satisfies the equivalence principle and makes space locally Minkowskian (that is, in suitable locally inertial coordinates, the metric is Minkowskian, and its first partial derivatives and the connection coefficients vanish).[28]

Einstein's equations[edit]


Main articles: Einstein field equations and Mathematics of general relativity
Having formulated the relativistic, geometric version of the effects of gravity, the question of gravity's source remains. In Newtonian gravity, the source is mass. In special relativity, mass turns out to be part of a more general quantity called the energy–momentum tensor, which includes bothenergy and momentum densities as well as stress (that is, pressure and shear).[29] Using the equivalence principle, this tensor is readily generalized to curved space-time. Drawing further upon the analogy with geometric Newtonian gravity, it is natural to assume that the field equationfor gravity relates this tensor and the Ricci tensor, which describes a particular class of tidal effects: the change in volume for a small cloud of test particles that are initially at rest, and then fall freely. In special relativity, conservation of energy–momentum corresponds to the statement that the energy–momentum tensor is divergence-free. This formula, too, is readily generalized to curved spacetime by replacing partial derivatives with their curved-manifold counterparts, covariant derivatives studied indifferential geometry. With this additional condition—the covariant divergence of the energy–momentum tensor, and hence of whatever is on the other side of the equation, is zero— the simplest set of equations are what are called Einstein's (field) equations:[/ltr]

Einstein's field equations
Quantum Field Theory III 9fe9108d287a6994c9f123644a43f3ba
[ltr]

On the left-hand side is the Einstein tensor, a specific divergence-free combination of the Ricci tensor Quantum Field Theory III B47576ad049849a296a27e80d56d6c25 and the metric. Where Quantum Field Theory III 66c608e07a31c3051ede7e0ff8218f5f is symmetric. In particular,
Quantum Field Theory III 001a96a2a12c7f6ff64ea490ee063f8f
is the curvature scalar. The Ricci tensor itself is related to the more general Riemann curvature tensor as
Quantum Field Theory III 898c45207527c26875f8b5c548f90a80
On the right-hand side, Quantum Field Theory III B2d92ed610f92cabe8692810d383f0fb is the energy–momentum tensor. All tensors are written in abstract index notation.[30] Matching the theory's prediction to observational results for planetary orbits (or, equivalently, assuring that the weak-gravity, low-speed limit is Newtonian mechanics), the proportionality constant can be fixed as κ = 8πG/c4, with G thegravitational constant and c the speed of light.[31] When there is no matter present, so that the energy–momentum tensor vanishes, the results are the vacuum Einstein equations,
Quantum Field Theory III 8611872f32299c95f3df7086fcec7d48
There are alternatives to general relativity built upon the same premises, which include additional rules and/or constraints, leading to different field equations. Examples are Brans–Dicke theoryteleparallelism, andEinstein–Cartan theory.[32]

Definition and basic applications[edit]



See also: Mathematics of general relativity and Physical theories modified by general relativity
The derivation outlined in the previous section contains all the information needed to define general relativity, describe its key properties, and address a question of crucial importance in physics, namely how the theory can be used for model-building.

Definition and basic properties[edit]



General relativity is a metric theory of gravitation. At its core are Einstein's equations, which describe the relation between the geometry of a four-dimensional, pseudo-Riemannian manifold representing spacetime, and the energy–momentum contained in that spacetime.[33] Phenomena that inclassical mechanics are ascribed to the action of the force of gravity (such as free-fallorbital motion, and spacecraft trajectories), correspond to inertial motion within a curved geometry of spacetime in general relativity; there is no gravitational force deflecting objects from their natural, straight paths. Instead, gravity corresponds to changes in the properties of space and time, which in turn changes the straightest-possible paths that objects will naturally follow.[34] The curvature is, in turn, caused by the energy–momentum of matter. Paraphrasing the relativist John Archibald Wheeler, spacetime tells matter how to move; matter tells spacetime how to curve.[35]
While general relativity replaces the scalar gravitational potential of classical physics by a symmetric rank-two tensor, the latter reduces to the former in certain limiting cases. For weak gravitational fields and slow speed relative to the speed of light, the theory's predictions converge on those of Newton's law of universal gravitation.[36]
As it is constructed using tensors, general relativity exhibits general covariance: its laws—and further laws formulated within the general relativistic framework—take on the same form in all coordinate systems.[37]Furthermore, the theory does not contain any invariant geometric background structures, i.e. it is background independent. It thus satisfies a more stringent general principle of relativity, namely that the laws of physics are the same for all observers.[38] Locally, as expressed in theequivalence principle, spacetime is Minkowskian, and the laws of physics exhibit local Lorentz invariance.[39]

Model-building[edit]


The core concept of general-relativistic model-building is that of a solution of Einstein's equations. Given both Einstein's equations and suitable equations for the properties of matter, such a solution consists of a specific semi-Riemannian manifold (usually defined by giving the metric in specific coordinates), and specific matter fields defined on that manifold. Matter and geometry must satisfy Einstein's equations, so in particular, the matter's energy–momentum tensor must be divergence-free. The matter must, of course, also satisfy whatever additional equations were imposed on its properties. In short, such a solution is a model universe that satisfies the laws of general relativity, and possibly additional laws governing whatever matter might be present.[40]
Einstein's equations are nonlinear partial differential equations and, as such, difficult to solve exactly.[41] Nevertheless, a number of exact solutionsare known, although only a few have direct physical applications.[42] The best-known exact solutions, and also those most interesting from a physics point of view, are the Schwarzschild solution, the Reissner–Nordström solution and the Kerr metric, each corresponding to a certain type of black hole in an otherwise empty universe,[43] and the Friedmann–Lemaître–Robertson–Walker and de Sitter universes, each describing an expanding cosmos.[44] Exact solutions of great theoretical interest include the Gödel universe (which opens up the intriguing possibility of time travel in curved spacetimes), the Taub-NUT solution (a model universe that ishomogeneous, but anisotropic), and anti-de Sitter space (which has recently come to prominence in the context of what is called the Maldacena conjecture).[45]
Given the difficulty of finding exact solutions, Einstein's field equations are also solved frequently by numerical integration on a computer, or by considering small perturbations of exact solutions. In the field of numerical relativity, powerful computers are employed to simulate the geometry of spacetime and to solve Einstein's equations for interesting situations such as two colliding black holes.[46] In principle, such methods may be applied to any system, given sufficient computer resources, and may address fundamental questions such as naked singularities. Approximate solutions may also be found by perturbation theories such as linearized gravity[47]and its generalization, the post-Newtonian expansion, both of which were developed by Einstein. The latter provides a systematic approach to solving for the geometry of a spacetime that contains a distribution of matter that moves slowly compared with the speed of light. The expansion involves a series of terms; the first terms represent Newtonian gravity, whereas the later terms represent ever smaller corrections to Newton's theory due to general relativity.[48] An extension of this expansion is theparametrized post-Newtonian (PPN) formalism, which allows quantitative comparisons between the predictions of general relativity and alternative theories.[49]

Consequences of Einstein's theory[edit]


General relativity has a number of physical consequences. Some follow directly from the theory's axioms, whereas others have become clear only in the course of the ninety years of research that followed Einstein's initial publication.

Gravitational time dilation and frequency shift[edit]


Main article: Gravitational time dilation[/ltr]

Quantum Field Theory III 220px-Gravitational_red-shifting


Schematic representation of the gravitational redshift of a light wave escaping from the su***ce of a massive body

[ltr]
Assuming that the equivalence principle holds,[50] gravity influences the passage of time. Light sent down into a gravity wellis blueshifted, whereas light sent in the opposite direction (i.e., climbing out of the gravity well) isredshifted; collectively, these two effects are known as the gravitational frequency shift. More generally, processes close to a massive body run more slowly when compared with processes taking place farther away; this effect is known as gravitational time dilation.[51]
Gravitational redshift has been measured in the laboratory[52] and using astronomical observations.[53] Gravitational time dilation in the Earth's gravitational field has been measured numerous times using atomic clocks,[54] while ongoing validation is provided as a side effect of the operation of the Global Positioning System (GPS).[55] Tests in stronger gravitational fields are provided by the observation of binary pulsars.[56] All results are in agreement with general relativity.[57] However, at the current level of accuracy, these observations cannot distinguish between general relativity and other theories in which the equivalence principle is valid.[58]

Light deflection and gravitational time delay[edit]


Main articles: Kepler problem in general relativityGravitational lensand Shapiro delay[/ltr]

Quantum Field Theory III 170px-Light_deflection


Deflection of light (sent out from the location shown in blue) near a compact body (shown in gray)

[ltr]
General relativity predicts that the path of light is bent in a gravitational field; light passing a massive body is deflected towards that body. This effect has been confirmed by observing the light of stars or distant quasars being deflected as it passes the Sun.[59]
This and related predictions follow from the fact that light follows what is called a light-like or null geodesic—a generalization of the straight lines along which light travels in classical physics. Such geodesics are the generalization of the invariance oflightspeed in special relativity.[60] As one examines suitable model spacetimes (either the exterior Schwarzschild solution or, for more than a single mass, the post-Newtonian expansion),[61] several effects of gravity on light propagation emerge. Although the bending of light can also be derived by extending theuniversality of free fall to light,[62] the angle of deflection resulting from such calculations is only half the value given by general relativity.[63]
Closely related to light deflection is the gravitational time delay (or Shapiro delay), the phenomenon that light signals take longer to move through a gravitational field than they would in the absence of that field. There have been numerous successful tests of this prediction.[64] In the parameterized post-Newtonian formalism (PPN), measurements of both the deflection of light and the gravitational time delay determine a parameter called γ, which encodes the influence of gravity on the geometry of space.[65]
[/ltr]


由一星于2014-09-11, 16:14进行了最后一次编辑,总共编辑了1次
一星
一星

帖子数 : 3787
注册日期 : 13-08-07

返回页首 向下

Quantum Field Theory III Empty 回复: Quantum Field Theory III

帖子 由 一星 2014-09-11, 15:58

Gravitational waves[edit]


Main article: Gravitational wave

Quantum Field Theory III 220px-Gravwav


Ring of test particles influenced by gravitational wave


One of several analogies between weak-field gravity andelectromagnetism is that, analogous to electromagnetic waves, there are gravitational waves: ripples in the metric of spacetime that propagate at thespeed of light.[66] The simplest type of such a wave can be visualized by its action on a ring of freely floating particles. A sine wave propagating through such a ring towards the reader distorts the ring in a characteristic, rhythmic fashion (animated image to the right).[67] Since Einstein's equations are non-linear, arbitrarily strong gravitational waves do not obey linear superposition, *** their description difficult. However, for weak fields, a linear approximation can be made. Such linearized gravitational waves are sufficiently accurate to describe the exceedingly weak waves that are expected to arrive here on Earth from far-off cosmic events, which typically result in relative distances increasing and decreasing by Quantum Field Theory III 33bbf8fb0c2991c3d9ff22ea913c51cf or less. Data analysis methods routinely make use of the fact that these linearized waves can be Fourier decomposed.[68]

Some exact solutions describe gravitational waves without any approximation, e.g., a wave train traveling through empty space[69] or so-called Gowdy universes, varieties of an expanding cosmos filled with gravitational waves.[70] But for gravitational waves produced in astrophysically relevant situations, such as the merger of two black holes,numerical methods are presently the only way to construct appropriate models.[71]

Orbital effects and the relativity of direction[edit]


Main article: Kepler problem in general relativity

General relativity differs from classical mechanics in a number of predictions concerning orbiting bodies. It predicts an overall rotation (precession) of planetary orbits, as well as orbital decay caused by the emission of gravitational waves and effects related to the relativity of direction.

Precession of apsides[edit]


Quantum Field Theory III 220px-Relativistic_precession.svg


Newtonian (red) vs. Einsteinian orbit (blue) of a lone planet orbiting a star


In general relativity, the apsides of any orbit (the point of the orbiting body's closest approach to the system's center of mass) willprecess—the orbit is not an ellipse, but akin to an ellipse that rotates on its focus, resulting in a rose curve-like shape (see image). Einstein first derived this result by using an approximate metric representing the Newtonian limit and treating the orbiting body as atest particle. For him, the fact that his theory gave a straightforward explanation of the anomalous perihelion shift of the planet Mercury, discovered earlier by Urbain Le Verrier in 1859, was important evidence that he had at last identified the correct form of thegravitational field equations.[72]

The effect can also be derived by using either the exact Schwarzschild metric (describing spacetime around a spherical mass)[73] or the much more general post-Newtonian formalism.[74] It is due to the influence of gravity on the geometry of space and to the contribution of self-energy to a body's gravity (encoded in the nonlinearity of Einstein's equations).[75]Relativistic precession has been observed for all planets that allow for accurate precession measurements (Mercury, Venus, and Earth),[76] as well as in binary pulsar systems, where it is larger by five orders of magnitude.[77]

Orbital decay[edit]


Quantum Field Theory III 220px-Psr1913%2B16-weisberg_en


Orbital decay for PSR1913+16: time shift in seconds, tracked over three decades.[78]


According to general relativity, abinary system will emit gravitational waves, thereby losing energy. Due to this loss, the distance between the two orbiting bodies decreases, and so does their orbital period. Within the Solar System or for ordinary double stars, the effect is too small to be observable. This is not the case for a close binary pulsar, a system of two orbitingneutron stars, one of which is apulsar: from the pulsar, observers on Earth receive a regular series of radio pulses that can serve as a highly accurate clock, which allows precise measurements of the orbital period. Because neutron stars are very compact, significant amounts of energy are emitted in the form of gravitational radiation.[79]

The first observation of a decrease in orbital period due to the emission of gravitational waves was made by Hulse and Taylor, using the binary pulsarPSR1913+16 they had discovered in 1974. This was the first detection of gravitational waves, albeit indirect, for which they were awarded the 1993Nobel Prize in physics.[80] Since then, several other binary pulsars have been found, in particular the double pulsar PSR J0737-3039, in which both stars are pulsars.[81]

Geodetic precession and frame-dragging[edit]


Main articles: Geodetic precession and Frame dragging

Several relativistic effects are directly related to the relativity of direction.[82] One is geodetic precession: the axis direction of a gyroscopein free fall in curved spacetime will change when compared, for instance, with the direction of light received from distant stars—even though such a gyroscope represents the way of keeping a direction as stable as possible ("parallel transport").[83] For the Moon–Earth system, this effect has been measured with the help of lunar laser ranging.[84] More recently, it has been measured for test masses aboard the satellite Gravity Probe B to a precision of better than 0.3%.[85][86]

Near a rotating mass, there are so-called gravitomagnetic or frame-dragging effects. A distant observer will determine that objects close to the mass get "dragged around". This is most extreme for rotating black holeswhere, for any object entering a zone known as the ergosphere, rotation is inevitable.[87] Such effects can again be tested through their influence on the orientation of gyroscopes in free fall.[88] Somewhat controversial tests have been performed using the LAGEOS satellites, confirming the relativistic prediction.[89] Also the Mars Global Surveyor probe around Mars has been used.[90][91]

Astrophysical applications[edit]


Gravitational lensing[edit]


Main article: Gravitational lensing

Quantum Field Theory III 220px-Einstein_cross


Einstein cross: four images of the same astronomical object, produced by a gravitational lens


The deflection of light by gravity is responsible for a new class of astronomical phenomena. If a massive object is situated between the astronomer and a distant target object with appropriate mass and relative distances, the astronomer will see multiple distorted images of the target. Such effects are known asgravitational lensing.[92] Depending on the configuration, scale, and mass distribution, there can be two or more images, a bright ring known as an Einstein ring, or partial rings called arcs.[93] The earliest example was discovered in 1979;[94] since then, more than a hundred gravitational lenses have been observed.[95] Even if the multiple images are too close to each other to be resolved, the effect can still be measured, e.g., as an overall brightening of the target object; a number of such "microlensing events" have been observed.[96]

Gravitational lensing has developed into a tool of observational astronomy. It is used to detect the presence and distribution of dark matter, provide a "natural telescope" for observing distant galaxies, and to obtain an independent estimate of the Hubble constant. Statistical evaluations of lensing data provide valuable insight into the structural evolution ofgalaxies.[97]

Gravitational wave astronomy[edit]


Main articles: Gravitational wave and Gravitational wave astronomy

Quantum Field Theory III 180px-LISA


Artist's impression of the space-borne gravitational wave detector LISA


Observations of binary pulsars provide strong indirect evidence for the existence of gravitational waves (see Orbital decay, above). However, gravitational waves reaching us from the depths of the cosmos have not been detected directly. Such detection is a major goal of current relativity-related research.[98] Several land-based gravitational wave detectorsare currently in operation, most notably the interferometric detectors GEO 600,LIGO (two detectors), TAMA 300 andVIRGO.[99] Various pulsar timing arraysare using millisecond pulsars to detect gravitational waves in the 10−9 to 10−6Hertz frequency range, which originate from binary supermassive blackholes.[100] European space-based detector, eLISA / NGO, is currently under development,[101] with a precursor mission (LISA Pathfinder) due for launch in 2015.[102]

Observations of gravitational waves promise to complement observations in the electromagnetic spectrum.[103] They are expected to yield information about black holes and other dense objects such as neutron stars and white dwarfs, about certain kinds of supernova implosions, and about processes in the very early universe, including the signature of certain types of hypothetical cosmic string.[104]

Black holes and other compact objects[edit]


Main article: Black hole

Whenever the ratio of an object's mass to its radius becomes sufficiently large, general relativity predicts the formation of a black hole, a region of space from which nothing, not even light, can escape. In the currently accepted models of stellar evolutionneutron stars of around 1.4 solar masses, and stellar black holes with a few to a few dozen solar masses, are thought to be the final state for the evolution of massive stars.[105]Usually a galaxy has one supermassive black hole with a few million to a few billion solar masses in its center,[106] and its presence is thought to have played an important role in the formation of the galaxy and larger cosmic structures.[107]

Quantum Field Theory III 220px-Star_collapse_to_black_hole


Simulation based on the equations of general relativity: a star collapsing to form a black hole while emitting gravitational waves


Astronomically, the most important property of compact objects is that they provide a supremely efficient mechanism for converting gravitational energy into electromagnetic radiation.[108]Accretion, the falling of dust or gaseous matter onto stellar orsupermassive black holes, is thought to be responsible for some spectacularly luminous astronomical objects, notably diverse kinds of active galactic nuclei on galactic scales and stellar-size objects such as microquasars.[109] In particular, accretion can lead torelativistic jets, focused beams of highly energetic particles that are being flung into space at almost light speed.[110] General relativity plays a central role in modelling all these phenomena,[111] and observations provide strong evidence for the existence of black holes with the properties predicted by the theory.[112]

Black holes are also sought-after targets in the search for gravitational waves (cf. Gravitational waves, above). Merging black hole binaries should lead to some of the strongest gravitational wave signals reaching detectors here on Earth, and the phase directly before the merger ("chirp") could be used as a "standard candle" to deduce the distance to the merger events–and hence serve as a probe of cosmic expansion at large distances.[113]The gravitational waves produced as a stellar black hole plunges into a supermassive one should provide direct information about the supermassive black hole's geometry.[114]



Cosmology[edit]


Quantum Field Theory III 220px-Lensshoe_hubble


This blue horseshoe is a distant galaxy that has been magnified and warped into a nearly complete ring by the strong gravitational pull of the massive foreground luminous red galaxy.


Main article: Physical cosmology

The current models of cosmology are based on Einstein's field equations, which include thecosmological constant Λ since it has important influence on the large-scale dynamics of the cosmos,
Quantum Field Theory III 518812acab136ddf18e36e6c288eff50
where Quantum Field Theory III F6a5a76efcf11af3b6e5fef03a7024b1 is the spacetime metric.[115] Isotropic and homogeneoussolutions of these enhanced equations, the Friedmann–Lemaître–Robertson–Walker solutions,[116] allow physicists to model a universe that has evolved over the past 14 billion years from a hot, early Big Bangphase.[117] Once a small number of parameters (for example the universe's mean matter density) have been fixed by astronomical observation,[118]further observational data can be used to put the models to the test.[119]Predictions, all successful, include the initial abundance of chemical elements formed in a period of primordial nucleosynthesis,[120] the large-scale structure of the universe,[121] and the existence and properties of a "thermal echo" from the early cosmos, the cosmic background radiation.[122]

Astronomical observations of the cosmological expansion rate allow the total amount of matter in the universe to be estimated, although the nature of that matter remains mysterious in part. About 90% of all matter appears to be so-called dark matter, which has mass (or, equivalently, gravitational influence), but does not interact electromagnetically and, hence, cannot be observed directly.[123] There is no generally accepted description of this new kind of matter, within the framework of known particle physics[124] or otherwise.[125] Observational evidence from redshift surveys of distantsupernovae and measurements of the cosmic background radiation also show that the evolution of our universe is significantly influenced by acosmological constant resulting in an acceleration of cosmic expansion or, equivalently, by a form of energy with an unusual equation of state, known as dark energy, the nature of which remains unclear.[126]

A so-called inflationary phase,[127] an additional phase of strongly accelerated expansion at cosmic times of around Quantum Field Theory III 02e083119bfd5a5d8bd9c3fde87d6cbd seconds, was hypothesized in 1980 to account for several puzzling observations that were unexplained by classical cosmological models, such as the nearly perfect homogeneity of the cosmic background radiation.[128] Recent measurements of the cosmic background radiation have resulted in the first evidence for this scenario.[129] However, there is a bewildering variety of possible inflationary scenarios, which cannot be restricted by current observations.[130] An even larger question is the physics of the earliest universe, prior to the inflationary phase and close to where the classical models predict the big bang singularity. An authoritative answer would require a complete theory of quantum gravity, which has not yet been developed[131] (cf. the section on quantum gravity, below).

Advanced concepts[edit]


Causal structure and global geometry[edit]


Main article: Causal structure

Quantum Field Theory III 220px-Penrose.svg


Penrose–Carter diagram of an infinite Minkowski universe


In general relativity, no material body can catch up with or overtake a light pulse. No influence from an event A can reach any other location X before light sent out at A to X. In consequence, an exploration of all light worldlines (null geodesics) yields key information about the spacetime's causal structure. This structure can be displayed using Penrose–Carter diagrams in which infinitely large regions of space and infinite time intervals are shrunk ("compactified") so as to fit onto a finite map, while light still travels along diagonals as in standard spacetime diagrams.[132]

Aware of the importance of causal structure, Roger Penrose and others developed what is known as global geometry. In global geometry, the object of study is not one particular solution (or family of solutions) to Einstein's equations. Rather, relations that hold true for all geodesics, such as the Raychaudhuri equation, and additional non-specific assumptions about the nature of matter (usually in the form of so-calledenergy conditions) are used to derive general results.[133]

Horizons[edit]


Main articles: Horizon (general relativity)No hair theorem and Black hole mechanics

Using global geometry, some spacetimes can be shown to contain boundaries called horizons, which demarcate one region from the rest of spacetime. The best-known examples are black holes: if mass is compressed into a sufficiently compact region of space (as specified in thehoop conjecture, the relevant length scale is the Schwarzschild radius[134]), no light from inside can escape to the outside. Since no object can overtake a light pulse, all interior matter is imprisoned as well. Passage from the exterior to the interior is still possible, showing that the boundary, the black hole's horizon, is not a physical barrier.[135]

Quantum Field Theory III 220px-Ergosphere.svg


The ergosphere of a rotating black hole, which plays a key role when it comes to extracting energy from such a black hole


Early studies of black holes relied on explicit solutions of Einstein's equations, notably the spherically symmetric Schwarzschild solution(used to describe a static black hole) and the axisymmetric Kerr solution (used to describe a rotating, stationary black hole, and introducing interesting features such as the ergosphere). Using global geometry, later studies have revealed more general properties of black holes. In the long run, they are rather simple objects characterized by eleven parameters specifying energy,linear momentumangular momentum, location at a specified time andelectric charge. This is stated by the black hole uniqueness theorems: "black holes have no hair", that is, no distinguishing marks like the hairstyles of humans. Irrespective of the complexity of a gravitating object collapsing to form a black hole, the object that results (having emittedgravitational waves) is very simple.[136]

Even more remarkably, there is a general set of laws known as black hole mechanics, which is analogous to the laws of thermodynamics. For instance, by the second law of black hole mechanics, the area of the event horizon of a general black hole will never decrease with time, analogous to the entropy of a thermodynamic system. This limits the energy that can be extracted by classical means from a rotating black hole (e.g. by thePenrose process).[137] There is strong evidence that the laws of black hole mechanics are, in fact, a subset of the laws of thermodynamics, and that the black hole area is proportional to its entropy.[138] This leads to a modification of the original laws of black hole mechanics: for instance, as the second law of black hole mechanics becomes part of the second law of thermodynamics, it is possible for black hole area to decrease—as long as other processes ensure that, overall, entropy increases. As thermodynamical objects with non-zero temperature, black holes should emit thermal radiation. Semi-classical calculations indicate that indeed they do, with the su***ce gravity playing the role of temperature in Planck's law. This radiation is known as Hawking radiation (cf. the quantum theory section, below).[139]

There are other types of horizons. In an expanding universe, an observer may find that some regions of the past cannot be observed ("particle horizon"), and some regions of the future cannot be influenced (event horizon).[140] Even in flat Minkowski space, when described by an accelerated observer (Rindler space), there will be horizons associated with a semi-classical radiation known as Unruh radiation.[141]

Singularities[edit]


Main article: Spacetime singularity

Another general feature of general relativity is the appearance of spacetime boundaries known as singularities. Spacetime can be explored by following up on timelike and lightlike geodesics—all possible ways that light and particles in free fall can travel. But some solutions of Einstein's equations have "ragged edges"—regions known as spacetime singularities, where the paths of light and falling particles come to an abrupt end, and geometry becomes ill-defined. In the more interesting cases, these are "curvature singularities", where geometrical quantities characterizing spacetime curvature, such as the Ricci scalar, take on infinite values.[142] Well-known examples of spacetimes with future singularities—where worldlines end—are the Schwarzschild solution, which describes a singularity inside an eternal static black hole,[143] or the Kerr solution with its ring-shaped singularity inside an eternal rotating black hole.[144] The Friedmann–Lemaître–Robertson–Walker solutions and other spacetimes describing universes have past singularities on which worldlines begin, namely Big Bang singularities, and some have future singularities (Big Crunch) as well.[145]

Given that these examples are all highly symmetric—and thus simplified—it is tempting to conclude that the occurrence of singularities is an artifact of idealization.[146] The famous singularity theorems, proved using the methods of global geometry, say otherwise: singularities are a generic feature of general relativity, and unavoidable once the collapse of an object with realistic matter properties has proceeded beyond a certain stage[147] and also at the beginning of a wide class of expanding universes.[148] However, the theorems say little about the properties of singularities, and much of current research is devoted to characterizing these entities' generic structure (hypothesized e.g. by the so-called BKL conjecture).[149] The cosmic censorship hypothesis states that all realistic future singularities (no perfect symmetries, matter with realistic properties) are safely hidden away behind a horizon, and thus invisible to all distant observers. While no formal proof yet exists, numerical simulations offer supporting evidence of its validity.[150]

Evolution equations[edit]


Main article: Initial value formulation (general relativity)

Each solution of Einstein's equation encompasses the whole history of a universe — it is not just some snapshot of how things are, but a whole, possibly matter-filled, spacetime. It describes the state of matter and geometry everywhere and at every moment in that particular universe. Due to its general covariance, Einstein's theory is not sufficient by itself to determine the time evolution of the metric tensor. It must be combined with a coordinate condition, which is analogous to gauge fixing in other field theories.[151]

To understand Einstein's equations as partial differential equations, it is helpful to formulate them in a way that describes the evolution of the universe over time. This is done in so-called "3+1" formulations, where spacetime is split into three space dimensions and one time dimension. The best-known example is the ADM formalism.[152] These decompositions show that the spacetime evolution equations of general relativity are well-behaved: solutions always exist, and are uniquely defined, once suitable initial conditions have been specified.[153] Such formulations of Einstein's field equations are the basis of numerical relativity.[154]
一星
一星

帖子数 : 3787
注册日期 : 13-08-07

返回页首 向下

Quantum Field Theory III Empty 回复: Quantum Field Theory III

帖子 由 一星 2014-09-11, 15:59

Global and quasi-local quantities[edit]


Main article: Mass in general relativity

The notion of evolution equations is intimately tied in with another aspect of general relativistic physics. In Einstein's theory, it turns out to be impossible to find a general definition for a seemingly simple property such as a system's total mass (or energy). The main reason is that the gravitational field—like any physical field—must be ascribed a certain energy, but that it proves to be fundamentally impossible to localize that energy.[155]

Nevertheless, there are possibilities to define a system's total mass, either using a hypothetical "infinitely distant observer" (ADM mass)[156] or suitable symmetries (Komar mass).[157] If one excludes from the system's total mass the energy being carried away to infinity by gravitational waves, the result is the so-called Bondi mass at null infinity.[158] Just as in classical physics, it can be shown that these masses are positive.[159]Corresponding global definitions exist for momentum and angular momentum.[160] There have also been a number of attempts to definequasi-local quantities, such as the mass of an isolated system formulated using only quantities defined within a finite region of space containing that system. The hope is to obtain a quantity useful for general statements about isolated systems, such as a more precise formulation of the hoop conjecture.[161]

Relationship with quantum theory[edit]


If general relativity is considered one of the two pillars of modern physics,quantum theory, the basis of understanding matter from elementary particles to solid state physics, is the other.[162] However, it is still an open question as to how the concepts of quantum theory can be reconciled with those of general relativity.

Quantum field theory in curved spacetime[edit]


Main article: Quantum field theory in curved spacetime

Ordinary quantum field theories, which form the basis of modernelementary particle physics, are defined in flat Minkowski space, which is an excellent approximation when it comes to describing the behavior of microscopic particles in weak gravitational fields like those found on Earth.[163] In order to describe situations in which gravity is strong enough to influence (quantum) matter, yet not strong enough to require quantization itself, physicists have formulated quantum field theories in curved spacetime. These theories rely on general relativity to describe a curved background spacetime, and define a generalized quantum field theory to describe the behavior of quantum matter within that spacetime.[164] Using this formalism, it can be shown that black holes emit a blackbody spectrum of particles known as Hawking radiation, leading to the possibility that they evaporate over time.[165] As briefly mentionedabove, this radiation plays an important role for the thermodynamics of black holes.[166]

Quantum gravity[edit]


Main article: Quantum gravity

See also: String theoryCanonical general relativityLoop quantum gravityCausal Dynamical Triangulations and Causal sets

The demand for consistency between a quantum description of matter and a geometric description of spacetime,[167] as well as the appearance ofsingularities (where curvature length scales become microscopic), indicate the need for a full theory of quantum gravity: for an adequate description of the interior of black holes, and of the very early universe, a theory is required in which gravity and the associated geometry of spacetime are described in the language of quantum physics.[168] Despite major efforts, no complete and consistent theory of quantum gravity is currently known, even though a number of promising candidates exist.[169]

Quantum Field Theory III 220px-Calabi_yau


Projection of a Calabi–Yau manifold, one of the ways ofcompactifying the extra dimensions posited by string theory


Attempts to generalize ordinary quantum field theories, used inelementary particle physics to describe fundamental interactions, so as to include gravity have led to serious problems. At low energies, this approach proves successful, in that it results in an acceptableeffective (quantum) field theory of gravity.[170] At very high energies, however, the result are models devoid of all predictive power ("non-renormalizability").[171]

Quantum Field Theory III 220px-Spin_network.svg


Simple spin network of the type used in loop quantum gravity


One attempt to overcome these limitations is string theory, a quantum theory not of point particles, but of minute one-dimensional extended objects.[172]The theory promises to be aunified description of all particles and interactions, including gravity;[173] the price to pay is unusual features such as six extra dimensions of space in addition to the usual three.[174] In what is called the second superstring revolution, it was conjectured that both string theory and a unification of general relativity andsupersymmetry known assupergravity[175] form part of a hypothesized eleven-dimensional model known as M-theory, which would constitute a uniquely defined and consistent theory of quantum gravity.[176]

Another approach starts with the canonical quantization procedures of quantum theory. Using the initial-value-formulation of general relativity (cf.evolution equations above), the result is the Wheeler–deWitt equation (an analogue of the Schrödinger equation) which, regrettably, turns out to be ill-defined.[177] However, with the introduction of what are now known asAshtekar variables,[178] this leads to a promising model known as loop quantum gravity. Space is represented by a web-like structure called aspin network, evolving over time in discrete steps.[179]

Depending on which features of general relativity and quantum theory are accepted unchanged, and on what level changes are introduced,[180] there are numerous other attempts to arrive at a viable theory of quantum gravity, some examples being dynamical triangulations,[181] causal sets,[182] twistor models[183] or the path-integral based models of quantum cosmology.[184]

All candidate theories still have major formal and conceptual problems to overcome. They also face the common problem that, as yet, there is no way to put quantum gravity predictions to experimental tests (and thus to decide between the candidates where their predictions vary), although there is hope for this to change as future data from cosmological observations and particle physics experiments becomes available.[185]

Current status[edit]


General relativity has emerged as a highly successful model of gravitation and cosmology, which has so far passed many unambiguous observational and experimental tests. However, there are strong indications the theory is incomplete.[186] The problem of quantum gravity and the question of the reality of spacetime singularities remain open.[187] Observational data that is taken as evidence for dark energy and dark matter could indicate the need for new physics.[188] Even taken as is, general relativity is rich with possibilities for further exploration. Mathematical relativists seek to understand the nature of singularities and the fundamental properties of Einstein's equations,[189] and increasingly powerful computer simulations (such as those describing merging black holes) are run.[190] The race for the first direct detection of gravitational waves continues,[191] in the hope of creating opportunities to test the theory's validity for much stronger gravitational fields than has been possible to date.[192] More than ninety years after its publication, general relativity remains a highly active area of research.[193]

See also[edit]





Introduction to mathematics of general relativity

From Wikipedia, the free encyclopedia



[ltr]This article is a non-technical introduction to the subject. For the main encyclopedia article, see Mathematics of general relativity.[/ltr]


[th]General relativity[/th][ltr]
The mathematics of general relativity are very complex. In Newton's theories of motions, an object's length and the rate of passage of time remain constant as it changes speed. As a result, many problems in Newtonian mechanics can be solved with algebra alone. In relativity, on the other hand, length, and the passage of time change as an object's speed approaches the speed of light. The additional variables greatly complicate calculations of an object's motion. As a result, relativity requires the use of vectors,tensorspseudotensors,curvilinear coordinates and many other complicated mathematical concepts.
For an introduction based on the specific physical example of particles orbiting a large mass in circular orbits, see Newtonian motivations for general relativity for a nonrelativistic treatment and Theoretical motivation for general relativity for a fully relativistic treatment.[/ltr]

[ltr]

Contents





  [hide[/ltr]

[ltr]

Vectors and tensors[edit]


Main articles: Euclidean vector and Tensor

Vectors[edit]

[/ltr]

Quantum Field Theory III 220px-Vector_by_Zureks.svg


Illustration of a typical vector.

[ltr]
In mathematicsphysics, andengineering, a Euclidean vector(sometimes called a geometric[1]or spatial vector,[2] or – as here – simply a vector) is a geometric object that has both a magnitude(or length) and direction. A vector is what is needed to "carry" the point A to the point B; the Latin word vector means "one who carries".[3] The magnitude of the vector is the distance between the two points and the direction refers to the direction of displacement from A to B. Many algebraic operations onreal numbers such as additionsubtractionmultiplication, and negationhave close analogues for vectors, operations which obey the familiar algebraic laws of commutativityassociativity, and distributivity.

Tensors[edit]

[/ltr]

Quantum Field Theory III 300px-Components_stress_tensor_cartesian.svg


Stress, a second-order tensor. Stress is here shown as a series of vectors on each side of the box

[ltr]
A tensor extends the concept of a vector to additional dimensions. A scalar, that is, a simple set of numbers without direction, would be shown on a graph as a point, a zero-dimensional object. A vector, which has a magnitude and direction, would appear on a graph as a line, which is a one-dimensional object. A tensor extends this concept to additional dimensions. A two dimensional tensor would be called a second order tensor. This can be viewed as a set of related vectors, moving in multiple directions on a plane.

Applications[edit]


Vectors are fundamental in the physical sciences. They can be used to represent any quantity that has both a magnitude and direction, such asvelocity, the magnitude of which is speed. For example, the velocity 5 meters per second upward could be represented by the vector (0,5) (in 2 dimensions with the positive y axis as 'up'). Another quantity represented by a vector is force, since it has a magnitude and direction. Vectors also describe many other physical quantities, such as displacement,accelerationmomentum, and angular momentum. Other physical vectors, such as the electric and magnetic field, are represented as a system of vectors at each point of a physical space; that is, a vector field.
Tensors also have extensive applications in physics:[/ltr]



[ltr]

Dimensions[edit]


In general relativity, four-dimensional vectors, or four-vectors, are required. These four dimensions are length, height, width and time. A "point" in this context would be an event, as it has both a location and a time. Similar to vectors, tensors in relativity require four dimensions. One example is the Riemann curvature tensor.

Coordinate transformation[edit]

[/ltr]


  • Quantum Field Theory III 200px-Transformation2polar_basis_vectors.svg
    A vector v, is shown with two coordinate grids, ex and er. In space, there is no clear coordinate grid to use. This means that the coordinate system changes based on the location and orientation of the observer. Observer exand er in this image are facing different directions.
     
  • Quantum Field Theory III 200px-Transformation2polar_contravariant_vector.svg
    Here we see that ex and er see the vector differently. The direction of the vector is the same. But to ex, the vector is moving to its left. To er, the vector is moving to its right.


[ltr]
In physics, as well as mathematics, a vector is often identified with a tuple, or list of numbers, which depend on some auxiliary coordinate system orreference frame. When the coordinates are transformed, for example by rotation or stretching, then the components of the vector also transform. The vector itself has not changed, but the reference frame has, so the components of the vector (or measurements taken with respect to the reference frame) must change to compensate.
The vector is called covariant or contravariant depending on how the transformation of the vector's components is related to the transformation of coordinates.[/ltr]


  • Contravariant vectors are "regular vectors" with units of distance (such as a displacement) or distance times some other unit (such as velocity or acceleration). For example, in changing units from meters to millimeters, a displacement of 1 m becomes 1000 mm.
  • Covariant vectors, on the other hand, have units of one-over-distance (typically such as gradient). For example, in changing again from meters to millimeters, a gradient of 1 K/m becomes 0.001 K/mm.


[ltr]
Coordinate transformation is important because relativity states that there is no one correct reference point in the universe. On earth, we use dimensions like north, east, and elevation, which are used throughout the entire planet. There is no such system for space. Without a clear reference grid, it becomes more accurate to describe the four dimensions as towards/away, left/right, up/down and past/future. As an example event, take the signing of the Declaration of Independence. To a modern observer on Mt Rainier looking east, the event is ahead, to the right, below, and in the past. However, to an observer in medieval England looking north, the event is behind, to the left, neither up nor down, and in the future. The event itself has not changed, the location of the observer has.

Oblique axes[edit]

[/ltr]

[ltr]An oblique coordinate system is one in which the axes are not necessarilyorthogonal to each other; that is, they meet at angles other than right angles.

Nontensors[edit]

[/ltr]
See also: Pseudotensor
[ltr]A nontensor is a tensor-like quantity that behaves like a tensor in the raising and lowering of indices, but that does not transform like a tensor under a coordinate transformation. For example, Christoffel symbolscannot be tensors themselves if the coordinates don't change in a linear way.

Curvilinear coordinates and curved spacetime[edit]

[/ltr]

Quantum Field Theory III 250px-Cassini-science-br


High-precision test of general relativity by the Cassini space probe (artist's impression): radio signals sent between the Earth and the probe (green wave) aredelayed by the warping of space and time(blue lines) due to the Sun's mass. That is, the Sun's mass causes the regular grid coordinate system (in blue) to distort and have curvature. The radio wave then follows this curvature and moves toward the Sun.

[ltr]
Curvilinear coordinates are coordinates in which the angles between axes can change from point-to-point. This means that rather than having a grid of straight lines, the grid instead has curvature.
A good example of this is the su***ce of the Earth. While maps frequently portray north, south, east and west as a simple square grid, that is not in fact the case. Instead, the longitude lines running north and south are curved and meet at the north pole. This is because the Earth is not flat, but instead round.
In general relativity, gravity has curvature effects on the four dimensions of the universe. A common analogy is placing a heavy object on a stretched out rubber sheet, causing the sheet to bend downward. This curves the coordinate system around the object, much like an object in the universe curves the coordinate system it sits in. The mathematics here are conceptually more complex than on Earth, as it results in 4 dimensions of curved coordinates instead of 3 as used to describe a curved 2D su***ce.

Parallel transport[edit]





Main article: Parallel transport[/ltr]

Quantum Field Theory III 450px-Parallel_displacement.svg


Example: Parallel displacement along a circle of a three-dimensional ball embedded in two dimensions. The circle of radius r is embedded in a two-dimensional space characterized by the coordinates Quantum Field Theory III 4df9fb3ddb36f50d8925c824a8564205 and Quantum Field Theory III 7db5e3ec92f18b8e39aa043575959617. The circle itself is characterized by coordinates Quantum Field Theory III 6aa0b456cc29eeaad0c9ac7ee134d1ee and Quantum Field Theory III 7734878eec1f16ee88099b432b86e29f in the two-dimensional space. The circle itself is one-dimensional and can be characterized by its arc length x. The coordinate y is related to the coordinate x through the relation Quantum Field Theory III Bc173e3f49f6347fcf3b0d79212b6f0e and Quantum Field Theory III 36a16cad3d458c2d81afba11851661a2. This gives Quantum Field Theory III 85d7da73aa27e73571fae549bb95807f and Quantum Field Theory III C4078804730cd34201c3edf0e94502bf In this case the metric is a scalar and is given by Quantum Field Theory III Caa04c78e671ee87830df93516872448 The interval is then Quantum Field Theory III 324df336707e8cd4f82c9072cd659641 The interval is just equal to the arc length as expected.
一星
一星

帖子数 : 3787
注册日期 : 13-08-07

返回页首 向下

Quantum Field Theory III Empty 回复: Quantum Field Theory III

帖子 由 一星 2014-09-12, 13:13

测地线[编辑]




Quantum Field Theory III 356px-Spherical_***.svg


3条测地线构成的球面三角形。在球面上,测地线是大圆

[ltr]
测地线又称大地线短程线数学上可视作直线在弯曲空间中的推广;在有度规定义存在之时,测地线可以定义为空间中两点的局域最短路径。测地线(geodesic)的名字来自对于地球尺寸与形状的大地测量学geodesy)。[/ltr]

[ltr]

目录



  [隐藏[/ltr]




[ltr]

三维空间中的曲面[编辑]



在大地线上,各点的主曲率方向均与该点上曲面法线相合。它在圆球面上为大圆弧,在平面上就是直线。在大地测量中,通常用大地线来代替法截线,作为研究和计算椭球面上各种问题。测地线是在一个曲面上,每一点处测地曲率均为零的曲线。

相关定理及推论[编辑]



曲面上非直线的曲线是测地线的充分必要条件是除了曲率为零的点以外,曲线的主法线重合于曲面的法线。
如果两曲面沿一曲线相切,并且此曲线是其中一个曲面的测地线,那么它也是另一个曲面的测地线。
过曲面上任一点,给定一个曲面的切方向,则存在唯一一条测地线切于此方向。
在适当的小范围内联结任意两点的测地线是最短线,所以测地线又称为短程线

微分几何的测地线[编辑]



在一个黎曼流形Quantum Field Theory III 69691c7bdcc3ce6d5d8a1361f22d04ac上,一条曲线Quantum Field Theory III B6c8dbfd133af85aacba01ff0d4bc3fc若符合常微分方程
Quantum Field Theory III D144cd3d29e7ed3e6b8e6a54bca0d90c
就称之为测地线。其中Quantum Field Theory III Fe3a83e41074834731743ab803cd4936Quantum Field Theory III 69691c7bdcc3ce6d5d8a1361f22d04ac上的列维-奇维塔联络。方程左边为曲线在流形上的加速度向量,所以方程是说测地线是在流形上加速度为零的曲线,也因此测地线必定是等速曲线。
以上方程用局部座标表示为
Quantum Field Theory III 9ec4496ced7ac4c1016e1f1fd1a447f7
其中Quantum Field Theory III 6418f69b46eb356a85b4125afc6bc9caQuantum Field Theory III 69691c7bdcc3ce6d5d8a1361f22d04ac黎曼度量克里斯托费尔符号

唯一性及存在性[编辑]



给定流形上一点Quantum Field Theory III 83878c91171338902e0fe0fb97a8c47a及点上一个非零的切向量Quantum Field Theory III C26156d3e1c5c3f75b7bb059e092ef88,因测地线方程是二阶常微分方程柯西-利普希茨定理指出存在区间Quantum Field Theory III F7f5fd49a5b7233d6a6ce59fc84b05e5,使得方程在此区间上存在唯一解
Quantum Field Theory III F777a7ae058244153421c0e9240abf75
满足初值条件Quantum Field Theory III C8f00db707fc5fa86398bc14d0670826,Quantum Field Theory III 8d053a62214c4c4783981ad8030cb7db。但因为方程是非线性的,故未必在实数线Quantum Field Theory III 0c95a37acc94ef8c093ce39c36e07886上存在解。
从上述方程解的唯一性,可知若两条测地线经过同一点,且在此点上有相同的切向量,则这两条测地线是同一条测地线中的两部份。
Quantum Field Theory III 7fab592a63121a749a125e0b9a639a0c是一条测地线,Quantum Field Theory III 6288d7285ef637d48bd4ac0abaf71d32。如果对起点Quantum Field Theory III 14d24c58b526ec5411bbbf285fbad6ae及起点的切向量Quantum Field Theory III C0abb4f1ed9df840d6833f0564094935改变得足够细微,则存在新的测地线符合新的初值条件,且仍然定义在Quantum Field Theory III 2c3d331bc98b44e71cb2aae9edadca7e上。这个结果用严格语言叙述为:
给定测地线Quantum Field Theory III 7fab592a63121a749a125e0b9a639a0c。在切丛Quantum Field Theory III Ac7a4fbd0d788d94f43454817e1c208c中存在Quantum Field Theory III C0abb4f1ed9df840d6833f0564094935的一个邻域Quantum Field Theory III 4c614360da93c0a041b22e537de151eb,使得对任何Quantum Field Theory III Aa52fd997d28deea603b58f43b48651a,都存在测地线Quantum Field Theory III B4203789fc0b87d42409a8954895b02c满足初值条件Quantum Field Theory III 1bdf2ad97c12b4f261772b87f0d7ea28
从这结果可以得出,如果Quantum Field Theory III 334de1ea38b615839e4ee6b65ee1b103是定义在有界开区间Quantum Field Theory III Dd7536794b63bf90eccfd37f9b147d7f上的测地线,对它的起点和此点上的切向量改变得足够细微的话,则存在一条新的测地线满足新的初值条件,并且定义在接近整条Quantum Field Theory III Dd7536794b63bf90eccfd37f9b147d7f上。[1]
如果对于任意初始条件Quantum Field Theory III C8f00db707fc5fa86398bc14d0670826,Quantum Field Theory III 8d053a62214c4c4783981ad8030cb7db都存在一条定义在整条实数线上的测地线Quantum Field Theory III Ab7bd0391566f136836bde58e74c5f8d,则称Quantum Field Theory III 69691c7bdcc3ce6d5d8a1361f22d04ac是测地完备的。霍普夫-里诺定理指出,若Quantum Field Theory III 69691c7bdcc3ce6d5d8a1361f22d04ac是一个完备度量空间,则Quantum Field Theory III 69691c7bdcc3ce6d5d8a1361f22d04ac是测地完备的。(Quantum Field Theory III 69691c7bdcc3ce6d5d8a1361f22d04ac上两点间的度量,是连接此两点的所有曲线的长度的最大下界。)

局部最短性[编辑]



在黎曼流形Quantum Field Theory III 69691c7bdcc3ce6d5d8a1361f22d04ac上连接两点之间的等速曲线,若其长度等于两点间的距离,即这曲线是两点间最短的曲线,那么这曲线必定是测地线。然而,连接两点间的测地线未必最短。比如在单位球面上,一条长度大于Quantum Field Theory III 522359592d78569a9eac16498aa7a087的测地线,不是连接这条线的两端点间的最短曲线。因为球面上的测地线都是大圆的弧,若测地线长度大于Quantum Field Theory III 522359592d78569a9eac16498aa7a087,那么测地线所在大圆上的另一条弧,其长度会小于Quantum Field Theory III 522359592d78569a9eac16498aa7a087,是连接这两点的最短测地线。
连接两点间最短测地线,也未必唯一。比如单位球面上两个对径点(即球面和一条直径的两个交点)之间,有无数条最短测地线相连。然而,流形上任何一点都存在一个邻域,使得该点和邻域上其他点之间,都有唯一的最短测地线相连(不计测地线的速度)。因此流形上任何测地线都是局部最短的。
对流形上一点Quantum Field Theory III 83878c91171338902e0fe0fb97a8c47a,一条从Quantum Field Theory III 83878c91171338902e0fe0fb97a8c47a出发的单位速的测地线Quantum Field Theory III Df7c519839d3ca22cacfe1b2bf978f04,考虑所有的Quantum Field Theory III 0535730078adc42f7af6fe8ce72846f3使得Quantum Field Theory III 4cc1893fa85f874e896bd04cb1811587,即是说Quantum Field Theory III 60638996369994a9abb2ef6006cdc45a是一条最短测地线。这集合可以是Quantum Field Theory III 2e67d9a4ad1dc9111da81573593e3713Quantum Field Theory III B14ef532a91b6a826c93e81adefa66cd。若是前者,称Quantum Field Theory III Bfb2b2f99169a05c4efdc3810ecdb6bbQuantum Field Theory III 83878c91171338902e0fe0fb97a8c47a沿着Quantum Field Theory III 334de1ea38b615839e4ee6b65ee1b103割点,那么对所有Quantum Field Theory III 04a66e3564ec59b96c1336ee0901d382,Quantum Field Theory III 60638996369994a9abb2ef6006cdc45a是从Quantum Field Theory III 83878c91171338902e0fe0fb97a8c47a点到Quantum Field Theory III Df7c519839d3ca22cacfe1b2bf978f04的唯一最短测地线;若是后者,则对所有Quantum Field Theory III 0535730078adc42f7af6fe8ce72846f3,Quantum Field Theory III 60638996369994a9abb2ef6006cdc45a都是Quantum Field Theory III 83878c91171338902e0fe0fb97a8c47a点到Quantum Field Theory III Df7c519839d3ca22cacfe1b2bf978f04的唯一最短测地线。Quantum Field Theory III 83878c91171338902e0fe0fb97a8c47a沿着全部从Quantum Field Theory III 83878c91171338902e0fe0fb97a8c47a出发的测地线的割点组成的集合,称为Quantum Field Theory III 83878c91171338902e0fe0fb97a8c47a割迹Quantum Field Theory III 4a8c3b277c934e455a861563c078120f

度量几何的测地线[编辑]



一般的度量空间Quantum Field Theory III 02129bb861061d1a052c592e2dc6b383中,测地线Quantum Field Theory III 334de1ea38b615839e4ee6b65ee1b103是从区间Quantum Field Theory III 11fadeac4d6b61685fec16911cf57c4d映射,使得对任何Quantum Field Theory III 4580b2c6ef891d108624855ae990527d,都存在区间Quantum Field Theory III 0f278ff0e8569e34b9b69f4fc854a1f0,使得Quantum Field Theory III Ff44570aca8241914870afbc310cdb85包含Quantum Field Theory III 6d523d2156a1f903c9cd55ab12627d5fQuantum Field Theory III Dd7536794b63bf90eccfd37f9b147d7f中一个邻域,并且对任何Quantum Field Theory III Dcc6f2be1065dbcb8b98396e38edff3b
Quantum Field Theory III 711b00a2fdd53d53c280d3381ba65c10
换言之,Quantum Field Theory III 3a854e134499034a70bfbdfa2d1c0271是连接其上任何两点的一条最短路线。[2]
如果一个度量空间任何两点都有测地线相连,称为测地度量空间。
度量空间上的测地线的性质,和微分几何有些不同:
两条测地线即使有部分线段重合,却未必属于同一条测地线。例如在Quantum Field Theory III 8c923e533839d95944b8528430091898上定义度量
Quantum Field Theory III 4d1c8f60083ebec856b0f0d616d82829
Quantum Field Theory III 2f3e35d839f5933caa3e303ae75b64bf是从(0,0)到(1,0)再到(1,1)的两条线段所组成,而Quantum Field Theory III F0eaadc2d5bf31a731efc6f5842788b1是从(0,0)到(2,0)的线段。这两条都是测地线,且在(0,0)到(1,0)一段重合,但明显不属同一条测地线,因为这两条线过了点(1,0)之后就分开。
一个测地度量空间中,在一点上未必存在一个邻域,使得该点其邻域其他点都有唯一的测地线。在上例的度量空间中,两点间如果两个座标都不同,则有无限多条测地线连接两点。例如从(0,0)到(2,1),以下都是连接这两点的最短测地线:任取一数Quantum Field Theory III F2b5955fff033cb4fd72ff89cb00233d
Quantum Field Theory III B48dd80747f978aeb23a275a35e5ad7e
就是先向右走到Quantum Field Theory III 40f0fbd51d00b8628ed4a68e81dcdc47,再向上走到Quantum Field Theory III 0ae4b485f7551a01d2882969a61d0d2a,再向右走到(2,1)。在任何一点的任何邻域中,和该点两个座标都不同的点有无数个,所以从该点到这些点之间,最短测地线都不是唯一。

参考[编辑]

[/ltr]


一星
一星

帖子数 : 3787
注册日期 : 13-08-07

返回页首 向下

Quantum Field Theory III Empty 回复: Quantum Field Theory III

帖子 由 一星 2014-09-14, 12:32

[ltr]微分几何是用微积分来研究几何。并与微分拓扑密切相关,这两个领域在物理上有很多应用,特别是在相对论中。他们共同成为微分流形的几何理论 - 这个理论也可以从动力系统的角度加以研究。

页面分类的主条目是微分几何

相关范畴[size=13][编辑]

[/ltr][/size]

[size][ltr]

子分类

本分类有以下14个子分类,共有14个子分类。
[/ltr][/size]

L


  • [ltr]► 李群‎ (6个分类, 40个页面)[/ltr]
  • [ltr]► 流形‎ (8个分类, 12个页面)[/ltr]
  • [ltr]► 黎曼几何‎ (4个分类, 34个页面)[/ltr]

Q


  • [ltr]► 曲線‎ (7个分类, 38个页面)[/ltr]
  • [ltr]► 曲面‎ (4个分类, 27个页面)[/ltr]

X


  • [ltr]► 纤维丛‎ (2个分类, 15个页面)[/ltr]
  • [ltr]► 辛幾何‎ (1个分类, 18个页面)[/ltr]

Z


  • [ltr]► 张量‎ (2个分类, 21个页面)[/ltr]




  • [ltr]► 旋量‎ (6个页面)[/ltr]



  • [ltr]► 联络‎ (1个分类, 15个页面)[/ltr]

[size][ltr]

分类“微分几何”中的页面

本分类中包含以下90个页面,共90个页面。
[/ltr][/size]

*


A


B


C


D


F


G


H


J


K


L


M


N


O


P


Q


S


W


W 续


X


Y


Z



















2个分类

[size]
个隐藏分类:
[/size]
[size]



高斯曲率[编辑]



[/size]
Quantum Field Theory III 220px-Gaussian_curvature

由左至右:负高斯曲率曲面(双曲面),零高斯曲率曲面(圆柱面),和正高斯曲率曲面(球面)。
[size][size][ltr]
微分几何中,曲面上一点的高斯曲率是该点主曲率κ1κ2的乘积。它是曲率的内在度量,也即,它的值只依赖于曲面上的距离如何测量,而不是曲面如何嵌入到空间。这个结果是高斯绝妙定理的主要内容。
用符号表示,高斯曲率K定义为
Quantum Field Theory III 6b725d6aa5950dd415f4ca42b33c076e.
也可以如下给出
Quantum Field Theory III E631ad7a7416ccb09496d4cd250c5470
其中Quantum Field Theory III 0301feada087017bbe4bcd34ca31cd00协变导数g度量张量
R3中的正规曲面的一点p,则高斯曲率为
Quantum Field Theory III 190dd32bce08a247ec75dec15e94be47
其中S形算子
关于高斯曲率的一个很有用的公式是用等温坐标中的拉普拉斯算子表达的刘维尔方程

[/ltr][/size][/size]
[ltr]

目录

  [隐藏[/ltr]


[size][size][ltr]

非形式化定义[编辑]

利用隐函数定理将曲面用二元函数f的图像来表示,并且假设点p为临界点,也即f在该点的梯度为0(这总是可以通过适当的刚体运动来实现)。然后p点的高斯曲率就是f在点p的黑塞矩阵(二阶导数组成的2x2矩阵)的行列式。这个定义只要用基本的微积分知识就可以理解杯底或者帽顶“对应”鞍点的区别。

总曲率[编辑]

[/ltr][/size][/size]
Quantum Field Theory III 220px-Hyperbolic_***.svg

负曲率曲面上的三角形三角之和小于平面三角形的三角之和。
[size][size][ltr]
曲面上某个区域的高斯曲率的曲面积分称为总曲率测地三角形的总曲率等于它的三角之和与Quantum Field Theory III 522359592d78569a9eac16498aa7a087的差别。正曲率曲面上的三角形三角之和大于Quantum Field Theory III 522359592d78569a9eac16498aa7a087,而负曲率曲面上的三角形三角之和小于Quantum Field Theory III 522359592d78569a9eac16498aa7a087。零曲率曲面上,三角之和正好为Quantum Field Theory III 522359592d78569a9eac16498aa7a087,譬如欧几里得平面
Quantum Field Theory III B074a2b9062acb731ad833f47db87d82
更一般的结果是高斯-博内定理

重要定理[编辑]

绝妙定理[编辑]

主条目:绝妙定理
高斯的绝妙定理断言曲面的高斯曲率由曲面上长度的测量本身决定。事实上,它完全由第一基本形式决定并且可以用第一基本形式及其一阶和二阶偏导数表达。等价地,嵌入在R3中的曲面的第二基本形式行列式也可以这样表达。定理的"绝妙"之处在于,虽然R3中的曲面S上的高斯曲率的定义明显依赖于曲面各点在空间中的定位,而高斯曲率本身只要曲面上的内在度量就可以决定,而与环境空间没有进一步的关联:它是一个内蕴不变量。精确地讲,高斯曲率在曲面的等度变换下保持不变。
在现代微分几何中,"曲面"抽象的看来是一个二维微分流形。将这个观点和曲面的经典理论联系起来的是将抽象曲面嵌入R3中,并用第一基本形式赋予黎曼度量。假设这个嵌入在R3中的像是曲面S局域等度就是R3中的开区域之间的微分同胚fU → V,限制到S ∩ U就是到自己的像的等度变换绝妙定理可以如下表述:
嵌入到R3的光滑曲面的高斯曲率在局域等度下不变。
例如圆柱面的高斯曲率为0,和"展开"后得到的平面是一样的。[1]另一方面,因为半径为R球面有正常数曲率R−2而平面有常数曲率0,这两个曲面不是等度的,即使局部也不行。因此即使是一部分球面的平面表示也会扭曲距离。所以没有测绘映射是完美的。

高斯-博内定理[编辑]

主条目:高斯-博内定理
高斯-博内定理将曲面的总曲率和它的欧拉示性数联系起来,并且给出了一个局部几何性质和全局拓扑性质的重要关联。

常曲率曲面[编辑]

[/ltr][/size][/size]

  • Minding定理(1839年)断言所有具有相同常曲率K的曲面局域等度。Minding的一个结果是所有曲率为0的曲面可以通过弯曲平面区域来构造。这样的曲面称为可展曲面。Minding也提出了有常正曲率的闭曲面是否刚性的问题。



  • Liebmann定理 (1900年)解决了Minding的问题。唯一常正曲率正则(C2)R3中的闭曲面是球面[2]



  • 希尔伯特定理 (1901年)断言在R3中不存在常负高斯曲率的完全解析(Cω)正则曲面。事实上,对于浸入到R3C2曲面也成立,但是对于C1-曲面却不成立。伪球面有常负高斯曲率,除了在其尖点[3]

[size][size][ltr]

其它公式[编辑]

[/ltr][/size][/size]
[size][size][ltr]
Quantum Field Theory III C16e4d08c58a418de3cd7b3f4bf7711b
[/ltr][/size][/size]

  • Brioschi公式只用第一基本形式给出高斯曲率:

[size][size][ltr]
Quantum Field Theory III 4f7f8433a31e8c66e18054660c298ab4
[/ltr][/size][/size]

  • 对于正交参数化,高斯曲率为:

[size][size][ltr]
Quantum Field Theory III 760a5c9b4cde1cf061d7ef77c0d4868e
[/ltr][/size][/size]

  • 高斯曲率是测地圆的周长和平面上的圆的周长之差的极限:

[size][size][ltr]
Quantum Field Theory III 5b4e53c1e7abb9c79df94314c76ad346
[/ltr][/size][/size]

  • 高斯曲率是测地圆的面积和平面上的圆的面积之差的极限:

[size][size][ltr]
Quantum Field Theory III 59adf83fda79162a16677287fd678b76
[/ltr][/size][/size]
[size][size][ltr]
Quantum Field Theory III 681e48dc961b4b1972b318e2463c344b

参考[编辑]

[/ltr][/size][/size][list=references]
[*]^ Porteous, I. R., Geometric Differentiation. Cambridge University Press, 1994. ISBN 0-521-39063-X

[*]^ Kühnel, Wolfgang. Differential Geometry: Curves - Su***ces - Manifolds. American Mathematical Society. 2006. ISBN 0821839888.

[*]^ Hilbert theorem. Springer Online Reference Works.

[*]^ Struik, Dirk. Lectures on Classical Differential Geometry. Courier Dover Publications. 1988. ISBN 0486656098.

[/list]
[size][size][ltr]

参看[编辑]

[/ltr][/size][/size]

[size][size]
4个分类
[/size][/size]

一星
一星

帖子数 : 3787
注册日期 : 13-08-07

返回页首 向下

Quantum Field Theory III Empty 回复: Quantum Field Theory III

帖子 由 一星 2015-02-13, 06:19

http://physics.stackexchange.com/questions/13870/gauge-symmetry-is-not-a-symmetry/29205#29205

Gauge symmetry is not a symmetry?

Gauge symmetry is not a symmetry?


I have read before in one of Seiberg's articles something like, that gauge symmetry is not a symmetry but a redundancy in our description, by introducing fake degrees of freedom to facilitate calculations.

Regarding this I have a few questions:

Why is it called a symmetry if it is not a symmetry? what about Noether theorem in this case? and the gauge groups U(1)...etc?
Does that mean, in principle, that one can gauge any theory (just by introducing the proper fake degrees of freedom)?
Are there analogs or other examples to this idea, of introducing fake degrees of freedom to facilitate the calculations or to build interactions, in classical physics? Is it like introducing the fictitious force if one insists on using Newton's 2nd law in a noninertial frame of reference?

按照现在人们对于量子场论的认识,所有的相互作用都可以从规范
原理导出来。每一个规范理论都对应一个规范对称群,这个群的元素对应于
一定的规范变换;比如,量子电动力学可以由一个规范对称群为U(1)的规范
理论来描述,其中规范场就是电磁场,U(1)群的群元素满足乘法交换性,所
以它是可交换的Abelian群。1954年,C.N.Yang并HR.Mills把规范场和规范变换
推广到群元素不满足乘法交换性的non.Abelian群,这就导致non.Abelian规范
场。non—Abelian规范场在诞生之初面临两大难题,一个是如何对场进行量子
化,另一个是如何赋予规范粒子以质量。对前者的研究一直贯穿着上个世纪
六十年代,其中Feynman、Fadeev、Popov$I]DeWitt等人做出了重要的贡献,其
成果就是所谓的路径积分量子化方法。对后者的研究一直伴随着对弱相互作
用的规范理论的处理,这个问题的解决得益于由对称性自发破缺(spontaneous
symmetry breaking)导致的Higgs机i昔lJ(Higgs Mechanism)[161。借助于这个机制,
以S.Glashow、S.Weinberg$[IA.Salam为代表的物理学家们最终建立了统一电磁
相互作用和弱相互作用的规范理论模型,臣PGlashow.Weinberg.Salam模型【17】。
该模型预言两个带电的(W士)和一个中性的(Zo)有质量的中间规范玻色子,它们
已经在实验上被发现;同时为了保证对称性自发破缺,Higgs机制还要求至少有
一个标量粒子—碰ggs粒子,但是到目前为止,还没有令人信服的实验上的证据
表明存在该粒子。
一星
一星

帖子数 : 3787
注册日期 : 13-08-07

返回页首 向下

返回页首


 
您在这个论坛的权限:
不能在这个论坛回复主题